Сохранен 535
https://2ch.hk/spc/res/280996.html
Домены arhivach.top и arhivach.site временно не функционируют! Используйте домен ARHIVACH.XYZ.
24 декабря Архивач восстановлен после серьёзной аварии. К сожалению, значительная часть сохранённых изображений и видео была потеряна. Подробности случившегося. Мы призываем всех неравнодушных помочь нам с восстановлением утраченного контента!

Тред тупых вопросов #48

 Аноним 09/04/16 Суб 15:34:11 #1 №280996 
14602052516750.jpg
14602052516751.jpg
14602052516772.jpg
14602052516793.jpg
Тред вопросов о жизни, Вселенной и всем таком.

Спрашиваем то, за что в других местах выдают путёвку в биореактор. Здесь анонимные ученые мирового уровня критически рассмотрят любые гениальные идеи и нарисованные в Paint схемы.

ХудСельсовет анонимных ученых напоминает, что кроме Paint существуют и другие программы GIMP, и для менее критического рассмотрения просим, тех кто не читают эту шапку, до оформлять ваши трехбуквенные вопросы, детским творчеством.

Совсем маленькие, могут разнообразить свой досуг здесь http://labs.minutelabs.io/Chaotic-Planets/

Прошлый тред https://2ch.hk/spc/res/274839.html
Аноним 09/04/16 Суб 15:39:13 #2 №281005 
>>280996 (OP)
Поясните суть 1 и 3 пикчи?
На 2 сфера Дайсона?
Аноним 09/04/16 Суб 15:42:34 #3 №281013 
Есть ли в космосе точка, в которой литр воды в вакууме не будет ни замерзать, ни испаряться? То есть вне атмосферы Земли.
Аноним 09/04/16 Суб 15:43:23 #4 №281016 
>>281013
Нет, для этого нужно давление.
Аноним 09/04/16 Суб 15:43:37 #5 №281017 
>>281005
пикчи найди два отличия - макаба нашла

на второй орбитальное кольцо, гибрид петли и лифта
Аноним 09/04/16 Суб 15:44:02 #6 №281019 
>>281005
Сферу то себе представляешь? Это кольцо Нивена.
Аноним 09/04/16 Суб 15:44:50 #7 №281020 
>>281019
А точно. Спасибо.

>>281016
В смысле?
Вообще, как я представляю, в космосе вода просто замёрзнет и разлетится на частицы - я правильно думаю?
Аноним 09/04/16 Суб 15:45:20 #8 №281023 
>>281005
на 1 и 3 кто-то тупил, думая что размер Луны может быть значительно больше или меньше, но эффект полного затмения все равно будет наблюдаться. В реальности это не так, даже краешек видимой фотосферы будет чрезвычайно ярким.
Аноним 09/04/16 Суб 15:48:10 #9 №281028 
>>281019
кольцо нивена, это ненужное переусложнение жилищного вопроса, а это просто транспортная система на орбите.
Аноним 09/04/16 Суб 15:50:23 #10 №281033 
>>281020
зависит от количества этой воды, и расстояния до солнца.
При благоприятных просто замерзнет в ледышку, часть испариться.
Близко к солнцу просто испариться все.
Аноним 09/04/16 Суб 16:07:14 #11 №281059 
>>280996 (OP)
Когда уже звездные сутки станут = 24часа?
Аноним 09/04/16 Суб 16:13:14 #12 №281066 
>>281033
А титановая болванка расплавится?
Аноним 09/04/16 Суб 17:17:05 #13 №281106 
>>281066
от формы зависит, нарисуй, посмотрим
Аноним 09/04/16 Суб 19:00:30 #14 №281165 
>>281005
>На 2 сфера Дайсона?
>на рисунке тонкое кольцо
>это сфера Дайсона?
>кольцо
>сфера
>а на рисунке кольцо...
>а спрашивает про сферу...
Да, это сфера Дайсона, вид с мкс.
Аноним 09/04/16 Суб 19:18:46 #15 №281174 
>>280996 (OP)
Вопрос о небесной механике.
Поясните про сферу действия, сферу тяготения и сферу Хилла. Отличия 1 и 3 я более менее понял, но нужно объяснение второй, ибо только она подходит для расчётов выведения КА на орбиту вокруг астероида Эрос.
Аноним 09/04/16 Суб 20:54:01 #16 №281213 
>>281174
>ибо только она подходит для расчётов выведения КА на орбиту вокруг астероида Эрос.
каким образом, поясни пожалуйста
Аноним 09/04/16 Суб 21:27:19 #17 №281229 
>>281213
Я сужу по данным миссии Near Shoemaker 1996 года. В момент посадки скорость аппарата относительно поверхности Эроса составила 1.9 м/с. При этом он больше месяца висел на круговой орбите вокруг астероида. Вот я и решил вычислить оптимальные параметры скорости и расстояния для того, чтобы КА смог стать искусственным спутником(пишу научный проект). Воспользовался понятиями сфер о которых писал выше. Использовал соответствующие формулы. И выяснилось следующие: Радиус сферы тяготения(область где астероид будет наиболее мощным притягивающим телом) составит всего 12 км, что очень близко к поверхности(неудивительно, сила тяготения у астероидов очень мала) да и аппрат будет вынужден вертется там со скорость 5 м/c. Радиус сферы Хилла же получился просто громадным(2270 км). Из-за влияния Солнца такая орбита будет очень нестабильна, поэтому отбрасываем. Осталась сфера действия, радиус которой составил 355 км (аппарат Near был чуть ближе к поверхности, что немного подтверждает расчёты). Потом посчитал 1 и 2 космические скорости для такого расстояния и получился вполне адекватный вариант! Для выхода на орбиту потребовалось бы приблизиться к астероиду на расстояние менее 355.1 км со скоростью менее 1.58 м/с. Вот как то так. Я пока не настолько образован, чтобы вертеться в научных кругах. Так что надежда пала на сосач. Численным методом я определи заменяя сферы действия, но полноценного понимаю о ней у меня по прежнему нет. Вот и спрашивал. Многовато букавок вышло.
Аноним 10/04/16 Вск 00:19:13 #18 №281358 
1. Что если это пришельцы вырубили японский телескоп, чтобы он не обнаружил их? Он мог бы обнаружить какие-то объекты, возможно корабли пришельцев?
Аноним 10/04/16 Вск 00:24:23 #19 №281359 
14602370639040.webm
>>281358
Аноним 10/04/16 Вск 00:27:51 #20 №281362 
>>281358
СК ПЗДЦ
Аноним 10/04/16 Вск 00:44:07 #21 №281364 
14602382470910.png
>>281362
>>281359
Че ржете? Вот вам вопрос - допустим в точке лагранжа 1 находится корабль пришельцев. Как мы его обнаружим, если он прячется в лучах Солнца?

Или корабль висит за орбитой Луны. Как мы его обнаружим? У нас есть военные радары в космосе для обнаружения космических кораблей?
Аноним 10/04/16 Вск 00:45:04 #22 №281365 
14602383041680.jpg
>>280996 (OP)
Чисто кукаретически, можно ли спустить на земляшку кубесат? Ну, чтобы по взрослому всё, на парашюте там, в назначенную точку. Чтобы можно было его найти опосля, а не абы куда. А с живыми организмами на борту? Запустить в космос ГОМУНКУЛА слизня какого-нибудь или там муравьёв. Осуществить, так сказать, личную космическую программу с выводом биологических организмов на околоземную орбиту и успешным их возвращением.
Аноним 10/04/16 Вск 00:49:29 #23 №281367 
>>281365
Ну будут нужны тормозной двигатель, теплощит и парашют. Вопрос в том, влезет ли всё это в формат кубсата, так? В принципе возможно. Маленький ТТРД, маленький щит... Больше всего места (и возможно веса) наверное потребует парашют.

(Если устроит чтобы он упал в рандомной точке, то можно и без ретроракеты.)
Аноним 10/04/16 Вск 00:52:07 #24 №281369 
>>281367
>>281365
Довольно таки просто как мне видится с дивана. Так как кубосат лёгкий, йоба-термощита не нужно.
Аноним 10/04/16 Вск 01:07:33 #25 №281382 
>>281358
>Что если это пришельцы вырубили японский телескоп
Если пришельцы - значит они как минимум смогли в пилотируемое межзвездное. А это в свою очередь значит, что мы для них обезьяны с палками. Если не захотят - мы их не обнаружим. Будут например транслировать на ближнюю к нам часть своего корабля картинку звездного неба позади. Хорошо транслировать, свою для каждого телескопа на земле, хайрез, 60 фпс.
А делать диверсии на телескопах это бред.
Аноним 10/04/16 Вск 01:08:27 #26 №281383 
14602397075580.png
>>281364
>Вот вам вопрос - допустим в точке лагранжа 1 находится корабль пришельцев. Как мы его обнаружим, если он прячется в лучах Солнца?
В L1 возможно обнаружат солнечные астрономы. За Солнцем ведется постоянное наблюдение, для предсказания погоды, состояния ионосферы и многого другого. Проблема в том, что L1 довольно размытое понятие, вокруг неё можно выписывать фигуры Лиссажу в очень широких пределах. Да и размеры играют роль, например такую мелочь как DSCOVR или WINS, болтающуюся в L1, они не увидят даже при прохождении на фоне солнца.

> Или корабль висит за орбитой Луны.
В L2 обнаружить ничего невозможно, естественно.

Да и вообще, тут околоземные астероиды едва видят https://www.youtube.com/watch?v=S_d-gs0WoUw , и ступени лунных ракет даже потеряли из виду, а ты про корабли какие-то. http://neo.jpl.nasa.gov/orbits/
Аноним 10/04/16 Вск 01:54:22 #27 №281397 
>>281229
>оптимальные параметры скорости и расстояния
оптимальные с точки зрения чего?

>Из-за влияния Солнца такая орбита будет очень нестабильна
Это не так. Примером такого дела является луна, с практической точки зрения орбита достаточно стабильна.
Ну и согласно вики, существуют как минимум 19 частных решений задачи 3 тел. /wiki/Задача_трёх_тел

Если я правильно всосал этот манускрипт http://pskgu.ru/ebooks/astro3/astro3_06_01.pdf , то:

Сфера действия и сфера тяготения - отличаются геометрически.

Сфера притяжения это тру сфера, на поверхности которой сила притяжения астероида равна силе притяжения солнца. И геометрический центр этой сферы не связан ни с одним из центров масс, хотя и находится на линии эти центры соединяющей.

Сфера действия это не тру сфера, с геометрическим центром в одном из центров масс.
И при массе астероида много меньше массы солнца - это сфера.
Физический смыл которой, что внутри этого объема тело можно принять на центр отсчета.
Но при этом это какието статические определения. как я понимаю, в которых можно развлекаться дифурами по времени

Сфера Хила это уже динамическое определение, с физическим смыслом, что вокруг чего крутится. На ней же находятся точки лагранжа. (и эт не те же точки равенства гравитационных сил, что в сфере притяжения)

И все возможные орбиты которые ты хочешь рассматривать лежат внутри сферы хила.
L1 от 1700 до 2700 км
Те на полторы тысячи км это вполне себе должна быть валидная орбита, если не переусложнять.
Аноним 10/04/16 Вск 01:56:43 #28 №281399 
>>281382
Это мемчик про суперразвитых пришельцев уже доебал. Если они умеют в межзвездное, то не значит, что у них все гладко и им все доступно. Можно спокойно представить пришельцев, которые терпят аварию после межзвездного путешествия и не могут восстановить какую-то технологию, деградируют, испытывают дефицит какого-то ресурса. Или у тебя только у людей случаются катастрофы, неудачи, непредвиденные повороты событий, а пришельцы как феи защищены от всего? Если они из мяса у них может выйти из строя система жизнеобеспечения. Если они полагаются на ИИ, он может сломаться и придется думать своими мозгами. Вот тогда то им и придется прятать корабль от телескопов людей, пока они занимаются починкой и собирают о нас сведения через Интернет.
Знаю, знаю это сюжет Глубины в небе. Но аномальные потери спутников всегда должны привлекать наше внимание.
Аноним 10/04/16 Вск 02:05:54 #29 №281403 
>>281399
>Это мемчик про суперразвитых пришельцев уже доебал. Если они умеют в межзвездное, то не значит, что у них все гладко и им все доступно.
разумно.
>Можно спокойно представить пришельцев, которые терпят аварию после межзвездного путешествия и не могут восстановить какую-то технологию, деградируют, испытывают дефицит какого-то ресурса.
хуита полная, зайди в сайфаич, я тебе там по хардкору поясню. Там как раз тредик есть, к примеру Межзвёздные полёты
Аноним 10/04/16 Вск 02:21:57 #30 №281405 
>>281383
В L2 возможно, хули нет-то. Просто никто особо туда специально не смотрит.
Аноним 10/04/16 Вск 03:05:51 #31 №281415 
>>281403
>разумно
Нет, это Хуита.
>Хуита
а вот тут ты аргументируй
> в сайфаич
Мы здесь обсуждаем, этот тред для общих вопросов самых разных тем.
>по хардкору
Сомневаюсь что ты способен, скорее веришь в божественных инопланетян, которые могут всё и никогда не испытывают недостатков.
>Межзвёздные полёты
Там вообще какие-то шизики тусуются, которые не умеют в аргументацию.
>курсив
Ты пидор что ле?
Аноним 10/04/16 Вск 03:57:34 #32 №281424 
>>281415
по предложенному вопросу затоплено уже пять тредов, это там где я участвовал, и это в слоудоске, раз в пять более слоу чем тут.
>пидор
хочешь пососать?
Аноним 10/04/16 Вск 11:21:07 #33 №281469 
>>281399
>Это мемчик про суперразвитых пришельцев уже доебал.
Никаких мемчиков, я отвечал на тупой вопрос про "пришельцы ломают телескопы". Бро, ну а что ты хотел: тупой вопрос - тупой ответ.
Конечно, по уму на фразу "пришельцы ломают телескопы" надо просто посылать на хуй. Потому что тут без вариантов насмотревшийся фильмов долбан.
Аноним 10/04/16 Вск 11:26:04 #34 №281472 
>>281424
>по предложенному вопросу затоплено уже пять тредов
Видимо реальный космос слишком скучен для активного обсуждения. Предложи, если знаешь вопросы интереснее.
Аноним 10/04/16 Вск 12:26:33 #35 №281499 
14602803938930.png
14602803938931.png
>>281397
Оптимальные с точки зрения расчётов тормозного импульса. Хотя слово оптимальные тут вряд ли подойдёт, ведь скорость в афелии эллиптической орбиты(гелиоцентрическая система) составит 22.49 км\с(!) Колоссальная разница по сравнению с той скоростью, которая была получена при расчёте сферы действия. Потребуется не хилый импульс, чтобы тормознуть 800-килограмовую бандуру с таких скоростей до 1.58 м\с. Но других вариантов у меня увы нет, так что надеюсь жури сжалится надо мной и сочтёт эти приближённые расчёты правильными. Тем более учитывая, что на данном этапе обучения я не владею большим количеством знаний. Ну а насчёт сфер. Орбита Луны действительно стабильна, при том, что Солнце притягивает её сильнее в 2,2 раза чем Земля. Но с КА,думаю,тут другая ситуация. Он не так массивен как Луна и соответственно его собственная сила тяготения бесконечно мала, из этого вытекает, что дальше 400 км Солнце будет по прежнему главным гравитационным телом, и скорее всего стабильной орбиты нам не видать. А астероид не Земля, его влиянием на больших расстояниях можно вообще пренебречь. В предисловии я на всякий случай указал что мы рассматриваем задачу двух тел(участок Земля- Солнце и участок Солнце - Эрос). Так что я решил в конечном итоге попытаться понять эти сферы на пальцах. Сфера тяготения - область в которой рассматриваемое тело является наиболее "притягательным" несмотря на наличие более массивных тел(как правило границы сферы довольно близки к телу). Сфера Хилла - большая область, форма которой на самом деле весьма нестабильна. Пространство вокруг астрономического объекта в котором он способен удерживать свой спутник несмотря на притяжение объекта, вокруг которого обращается сам(как в ситуации с Луной и Солнцем о которой я писал выше). Так что, учитывая твои рассуждения о различном направлении на центры масс скорее всего всё дело в сильном различии масс КА и Луны. Она может быть спутником Земли несмотря на то, что находится в сфере Хилла. А вот КА не думаю. Иу и последнее. Сфера действия. Я нашёл её более менее краткое объяснение такого вида: Областью действия, или сферой действия, меньшего тела m2 относительно большего тела m1 называют область пространства, в которой выполняется условие(смотри пикчу). Внутри сферы действия меньшее тело рассматривают в качестве центрального, а большее тело как возмущающее. Вне сферы действия за центральное принимают большее тело, а возмущающее меньшее. В ряде задач небесной механики оказывается возможным пренебречь в первом приближении влиянием на траекторию КА большего тела внутри сферы действия и меньшего тела вне этой сферы. Тогда внутри сферы действия движение КА будет происходить в центральном поле, создаваемом меньшим телом, а вне сферы действия - в центральном поле, создаваемом большим телом. Вот как то так, сложная шутка, эта небесная механика.
Аноним 10/04/16 Вск 14:10:53 #36 №281590 
>>281472
Обсуждение реального космоса базируется на фактическом материале, по определению, а знание фактического материала удел избранных.
Обсуждение же системы постулатов, вытащенных из головы более доступно и менее организованно. SciFy при этом предполагает накладывать физические законы этой реальности, но с этим уже начинаются проблемы, из-за несовпадения терминологии и малого пересечения баз знаний обсуждающих, и разной субъективной оценки весов значимости постулатов в обсуждаемой проблеме.

Обсуждение ближнего космоса более интересно с практической точки зрения, но даже избранным придется делать определенные допущения.

Предложения, ну давайте обсудим пик два опа, к примеру.
Аноним 10/04/16 Вск 15:31:37 #37 №281642 
Почему такая разница в цвете?
http://www.hq.nasa.gov/office/pao/History/alsj/a14/AS14-66-9232.jpg
http://mentallandscape.com/C_Zond07_1.jpg
Аноним 10/04/16 Вск 15:34:47 #38 №281647 
>>281642
Потому что вручную красили?
Аноним 10/04/16 Вск 15:35:44 #39 №281648 
>>281642
Во-первых, вторую явно сканировали на отъебись.
Во-вторых, вторая вроде с Зонда, а у наших с качеством фотографий вечно пиздец.
Аноним 10/04/16 Вск 15:37:32 #40 №281649 
>>281642
Олдрин рассказал, что цвет поверхности сильно зависит от того, под каким углом на неё смотришь относительно Солнца. По его словам, общего, основного цвета вообще не было. По мнению Армстронга, цвет поверхности в месте посадки был таким же, каким он воспринимался с орбиты при данном угле возвышения Солнца (около 10°). Он в основном серый, бледно-серый и слегка коричневатый, если смотреть в противоположном от Солнца направлении, и с более тёмными оттенками серого, если смотреть под углом 90° к Солнцу. 
Аноним 10/04/16 Вск 15:39:55 #41 №281650 
>>281648
Земля же нормально по цвету вышла, значит и Луна тоже в естественных цветах.
Аноним 10/04/16 Вск 15:41:43 #42 №281652 
14602921038010.jpg
14602921038071.jpg
>>281650
Хуйня, это плёнка, так что цвет тут передан не слишком точно.
Кстати, давай сравним. Слева - кропнутая фотография с Аполлона-17, 1972 год. Справа я кропнул приложенную тобой, Зонд-7, 1969 год.
Аноним 10/04/16 Вск 15:44:01 #43 №281657 
14602922412750.jpg
>>281652
Но что ещё любопытнее - на фото с Аполлона-17 (это классическая фотография "The Blue Marble") цвета исправленные, оригинальное фото имеет вот такой цвет.
Аноним 10/04/16 Вск 15:45:45 #44 №281659 
>>281652
Почти одинаковая цветопередача.
Вот еще фотки лунного грунта с луны-16
http://mentallandscape.com/C_Luna16_Sample2.jpg
http://mentallandscape.com/C_Luna16_Sample3.jpg
коричневенькие
Аноним 10/04/16 Вск 15:46:38 #45 №281660 
>>280996 (OP)
)))
Аноним 10/04/16 Вск 15:46:45 #46 №281661 
14602924058650.jpg
>>281657
А с современных камер выглядит вот так, фоточка с DISCOVR.
Аноним 10/04/16 Вск 15:48:07 #47 №281662 
>>281657
А где лежат такие оригинальн? Дай сцилку посмотреть.
Аноним 10/04/16 Вск 15:49:32 #48 №281663 
>>281642
>Почему такая разница в цвете?
Потому что тебе надо понять, что такое цвет в фото, и что его не бывает "естественного".
Аноним 10/04/16 Вск 15:51:55 #49 №281666 
>>281499
>Орбита Луны действительно стабильна, при том, что Солнце притягивает её сильнее в 2,2 раза чем Земля.
ок, это хорошо.
>что дальше 400 км Солнце будет по прежнему главным гравитационным телом
В этом нет ни какой проблемы.
Предположим вместо эроса там летает фаллос, массой 1кг, можно ли выйти на некоторую орбиту вокруг него.
Да можно. Надо просто выйти на орбиту с тем же периодом что и у фаллоса, таких орбит бесконечное множество. И пространственные конфигурации - относительное положение тел - будут периодически повторятся. И без учета других возмущающих факторов, с точки зрения фаллоса это будут замкнутые кривые.

Практика.
Орбита в той же плоскости что и фаллоса. Перицентр чуть ниже, апоцентр чуть выше или на оборот, алюс небольшой сдвиг по фазе, чтобы в узлах ничего не происходило.
Это будут две независимые орбиты, но фаллосу будет казаться что вокруг него что то вращается.
Или второй вариант - орбиты не лежат в одной плоскости апоцентр и перицентр практически одинаковы, небольшой сдвиг по фазе.

Или просто та же орбита с небольшим сдвигом по фазе.

Те любая орбита полученная небольшим искажением орбиты фаллоса(апоцентра, перицентра, фазы, наклонения), у которой тот же период(ну или близкий для практических целей) - может стать целевой для спутника нонейм.

Основной геморрой это выйти на такую орбиту и не перепутать ее с другим бесконечным множеством орбит, у которых период будет не совпадать.
С учетом отсутствия систем навигации, подходящих для этих целей, задача становится не совсем тривиальной.
И требуется время, для того чтобы понять, на какой орбите таки спутник находится, наблюдая положение спутника в системе отсчета связанной с эросом.

Те в принципе наименьшие затраты, это просто выход на орбиту эроса, до него будет миллион километров и спутник будет двигаться, с точки зрения эроса, по каким то кривым, ближе дальше, может даже вокруг.
Гравитация эроса будет возмущать эту возмущенную орбиту, также как и другие тела солнечной системы, но принципиально картина не изменится, кривые станут более замороченными.
К сожалению я не достаточно знаком с GMAT чтобы продемонстрировать это, а моя софтина этого пока не умеет показывать и мне лень ее сейчас пилить.

Была картинка из ксп треда, но я ее проебал.

Со стабильностью орбиты, если она за пол года не разойдется дальше чем нам требуется - она вполне нормальная, с практической точки зрения.
Те висишь рядом с эросом, палишь до него дистанцию, определяешь его собственное вращение, привязываешься к его топонимам, определяешь собственное вращение в системе координат эроса, наблюдаешь все это некоторое время, вычисляешь собственное возмущение орбиты относительно орбиты эроса, корректируешь себя согласно вычислениям, постепенно сводишь орбиты.
>Оптимальные с точки зрения расчётов тормозного импульса.
Те в системе двух тел ты просто оптимально выходишь на орбиту по которой летит эрос и все, на этом финита.
Както так чтоли.
Аноним 10/04/16 Вск 15:54:59 #50 №281669 
>>281662
Тут все хассельбладовские плёнки без обработки:
https://www.flickr.com/photos/projectapolloarchive/albums
Аноним 10/04/16 Вск 16:40:47 #51 №281707 
>>281648
>>281663
Вот тут, кстати Луна тоже не серая, а имеет коричневый цвет.
https://2ch.hk/spc/res/271614.html#272055
Аноним 10/04/16 Вск 16:48:58 #52 №281714 
Пасаны, тут с одногруппником спорили недавно на тему полетов. Он грил, что космос враждебный, там погибель и вообще от одной космической радиации пизда всему.
Я грил что нельзя отрицать вероятность того, что корабль смогут защитить от внешних угроз, в том числе и решить проблему с радиацией.
Вопрос такой, а что там еще из подводных камней кроме космического мусора, астероидов, комет, гравитационных полей из других планет?
Аноним 10/04/16 Вск 16:54:40 #53 №281717 
>>281714
Насчёт радиации он прав. При пилотируемых полетах дальше поясов ван Аллена единственный способ защиты это полёт в свинцовых гробах
Аноним 10/04/16 Вск 16:57:36 #54 №281718 
>>281714
солнечные вспышки, но это тоже сорт оф радиационная проблема.
Основное ты перечислил, гамма вспышки и прочие катаклизмы - одинаково опасны и для шипа и для земли.
Так что можешь идти пиздить своего одногруппника, этот обсос не имеет ни каких шансов против тебя.
Аноним 10/04/16 Вск 16:58:33 #55 №281719 
>>281717
сколько, в зивертах, быстро решительно!
Аноним 10/04/16 Вск 17:00:29 #56 №281720 
>>281719
От солнечной активности зависит.
Аноним 10/04/16 Вск 17:00:30 #57 №281721 
14602968307440.png
14602968307491.jpg
>>281714
Скажи ему, что "Пузырей" нацепляют и на всякий случай будет запас водки для снятия радиации. А уж на совсем всякий случай у "Монолита" попросить защиту или взять с собой Доктора.
Аноним 10/04/16 Вск 17:03:39 #58 №281723 
>>281719
https://ru.wikipedia.org/wiki/%D0%9A%D0%BE%D0%BB%D0%BE%D0%BD%D0%B8%D0%B7%D0%B0%D1%86%D0%B8%D1%8F_%D0%9C%D0%B0%D1%80%D1%81%D0%B0#.D0.A0.D0.B0.D0.B4.D0.B8.D0.B0.D1.86.D0.B8.D1.8F
Аноним 10/04/16 Вск 17:12:18 #59 №281727 
14602975389330.jpg
Как пожарить котлетки в невесомости?
Аноним 10/04/16 Вск 17:14:22 #60 №281728 
>>281718
Дак я ж за то, что когда-то сможем летать далеко, а приведя аргументы в виде >солнечные вспышки, радиация, космический мусор и т.д. я подтвержу его позицию, что туда соваться не стоит. Что бы пиздить его нужно иметь идеи про пути обхода или устранения препятствий для путешествий.
Аноним 10/04/16 Вск 17:31:55 #61 №281736 
14602987153460.jpg
>>281721
ГЕД АУД ОФ ХИР СТАЛКАР
Аноним 10/04/16 Вск 17:59:33 #62 №281752 
>>281723
мог бы и написать
>Ученые, по итогам восьмимесячного исследования, пришли к выводу, что участники пилотируемого полета к Марсу поглотят потенциально смертельную дозу космической радиации, свыше 1 зиверта ионизирующего излучения, две трети из которого путешественники получат во время полета к Марсу и Земле (около 1,8 миллизиверта излучения в день)
Мы конечно столько потратить на этот вопрос не можем.
Но, те 75 микрозивертов в час, стандартный фон 0.08-0.2 микрозивертов(в зависимости от местности, времени суток и прочего)

LD50 == 3-5 Sv http://rad-stop.ru/2-predelno-dopustimyie-i-letalnyie-dozyi

также https://geektimes.ru/post/181608/
>Для человека накопленная радиация в 1 Зиверт повышает риск раковых заболеваний на 5%. NASA позволяет своим астронавтам за свою карьеру, набирать не более 3% риска или 0,6 Зиверта. С учетом того, что на МКС ежедневная доза составляет до 1 мЗв, то предельный срок пребывания астронавтов на орбите ограничивается примерно 600 сутками за всю карьеру.

ИТОГО:
фон в сычевальне 0.08-0.2 mkSv/h
фон мкс 41mkSv/h
фон в космосе в два раза выше чем на мкс, те 80mkSv/h
Сколько получаете дозу - умножаете на время пребывания, в часах, больше 0.6 Sv лучше не получать.

Запоминаем эти числа: 0.1mkSv/h, 40mkSv/h, 80mkSv/h, 0.6Sv, 1Sv+5процентов, LD50 5Sv

Делаем скриншот, и когда захочется покукарекать о радиации в космосе, сразу скриншот.

Аноним 10/04/16 Вск 18:23:43 #63 №281756 
14603018236260.jpg
Зачем мкс? Пребывание в космосе нескольких людей оправдывает затраты на их содержание?
Аноним 10/04/16 Вск 19:08:30 #64 №281778 
14603045104570.jpg
>>281756
Базарю, хули им дома не сидится? Там жена наверно уже с любовником ебется, дети духовность без отца теряют. Могли бы и на море съездить и то лучше время б провели.
Аноним 10/04/16 Вск 23:29:48 #65 №281882 
Рассказывают что полетело на мкс и зачем, прессконференция.
https://www.youtube.com/watch?v=cq_Kl0IGHH0

Ктото спрашивал че они там делают.
Аноним 11/04/16 Пнд 00:03:18 #66 №281915 
>>281666
Я примерно понял идею. Спутник вращается вокруг Земли по эллипсу, затем мы даем разгонный импульс до второй космической и КА уходит из сферы действия Земли по гиперболе(которая характеризуется остаточной скоростью на бесконечности), что позволит нам сделать КА искуственным спутником Солнца. Подкорректировав его орбиту мы сможем соотнести ее с орбитой Эроса. Разве что нужно давать импульс в правильное время, ведь если направление импульса совпадает с направлением движения Земли то это безусловно плюс. Все должно пойти по плану, за исключением непредвиденных столкновений с килограмовым фалосом, который при такой кинетической энергии спомобен засадить прямо в душу. Не знаешь кстати, есть ли какие-то специализированные проги для рисования и вычисления орбит?(как раз тред для тупых вопросов) а то рисовать схемки в пеинте как то не айс.
Аноним 11/04/16 Пнд 00:12:16 #67 №281920 
>>281915
Можешь в КСП с рсс смоделировать довольно наглядно
Аноним 11/04/16 Пнд 00:12:20 #68 №281921 
>>281915
Я сам тут первый, хотя уже 12 стукнуло, так что пошел уже второй день в спейсаче, но шапку сходу зачитал. Там ссылка на что-то похожее есть. http://labs.minutelabs.io/Chaotic-Planets/
Аноним 11/04/16 Пнд 00:35:41 #69 №281939 
А нахуя это все? Я не про полеты в космос, а про технопрогресс вообще. Жить дольше, болезни лечить, сохранять окружающую среду и думать как сохранить цивилизацию через N столетий? Мы ж те черви из прометея, животные, которые руководствуются впрыском разных гормонов в мозг. По сути наркоманы, которые получают дозу от разных жизненных аспектов. Поцеловал тян, что тебе нравится - получил впрыск кайфа, нашел 100$ в кармане своего недруга на вписке - получил впрыск эндорфина, убежал от соседского питбуля через забор, а через пару дней отравил его, а сам слушая последние завывания умирающей псины получал впрыск кайфа.
Зачем нужна жизнь? Она есть просто потому, что так вышло, дак зачем ее поддерживать? Считаю, что людей нужно уничтожить биологическим оружием, а заместо людей оставить андроидов, которые были лишены человеческих дефектов, да и не жили б уже.
Аноним 11/04/16 Пнд 01:00:53 #70 №281947 
14603256533080.jpg
>>281939
>А нахуя это все?
Это всё для того, сынок, чтобы построить коммунизм.
Аноним 11/04/16 Пнд 01:47:11 #71 №281953 
>>281915
с програмками както не очень, погугли может повезет
есть http://gmatcentral.org/display/GW/GMAT+Wiki+Home
как бы предназначена для, но в минус идет то, что дохуя сложно както, и это было бы оправданно результатом, если бы она не была кастрированна по данным. Данных для обычного юзера достаточно, но сложность за эти результаты явно дохуя, при какойто хуевой базе знаний по этому софту, полумертвому форуму.

КСП на самом деле неплохой вариант, базу по орбитам он вполне отыгрывает, позволяет понять что это такое.
Особенно если задрочиться со стыковками со всякой фигней, посадками на луны и ловля планет и астероидов, и перехват астероидов - вообщем там полный набор.
Orbiter - под винду там вроде орбитальная механика на уровне.

Попадались самоделки, моделирования орбитальных грав вещей, видосы от них, но с самими программками както не фонтан, или я не там и не то и не так сильно искал.

> за исключением непредвиденных столкновений
это еще надо постараться, весь геморрой в этих перехватах. Япошки вон, чуть мимо планеты не промахнулись, едва вытянули.
Гравитационные маневры способ сильно сэкономить на дельте - вот там весь смак начинается, в плане экономии дельты, в лучших еврейских традициях.Ландау и его жена Лившиц

Хз я может попозже допишу свою приблуду, мне нужны траектории в L1б хочу понять как это выглядит с учетом основнх тел в солнечной системы. Пусть это будет не так круто и универсально как GMAT, но по крайней мере будет делать то что мне надо.
Те в принципе написание небольшой утилитки, которая пытается найти то что тебе нужно, это не такое плохое решение.
Обзываешь это численным моделированием, кричишь Монтекарло и телки текут.

Можно еще тут попытать народ http://space.stackexchange.com/
Там еще какой то раздел был, где можно было попытать, но если что там пошлют в правильном направлении, но в принципе не должны.
Аноним 11/04/16 Пнд 02:29:00 #72 №281957 
>>281939
Ты тупой. Это не тян целуешь, чтобы гормоны, а гормоны для того, чтобы ты целовал тян. Как можно делать такую очевидную ошибку? Всё равно что думать, будто смысл машин – в сжигании бензина, а процессоров – в рассеивании тепла. Мотивация существует для координации поведения, иначе бы мы просто переключались между разными сенсорными и когнитивными процессами, топчась на месте. Она реализована биохимически, как и всё остальное в людях. И что.
Аноним 11/04/16 Пнд 07:06:31 #73 №281977 
>>281399
Даже у потерпевшего крушение человека есть преимущество перед обезьяной. Потому что даже простой нож даст ему это преимущество. В зависимости от степени развития пришельцев их "деградация в результате катастрофы" может значительно превосходить все наши возможности. А может и нет конечно. Но помни про нож против обезьяны с палкой.
Аноним 11/04/16 Пнд 11:10:02 #74 №282006 
14603622024310.jpg
14603622024311.jpg
>>281977
Братан, нож это хуевый пример. Опуская недостаточную проникающую способность это человека в печень ширнул и готово у большой обезьяны толще слой тканей, а еще это приводит к главному козырю животных - реакция. Можно, конечно, найти какого-то Пака Сон Зы, который стрелы руками ловит, обучить ножевому бою и дать нож, тогда шанс будет, но тут вопрос: у обезьяны скиллы перманентные, человеку для их развития нужно тратить время и силы, а изначально они отсутствуют. Так что среднестатистическому человеку а еще с учетом что почти половина людей - это слабые шкуры обезьяна даст тебе пизды и с ножом.
Аноним 11/04/16 Пнд 11:16:21 #75 №282011 
>>281957
Ты пишешь из точки зрения личности или точки зрения биологического организма? Мотивация нужна для личности, что бы помочь организму - в будущем заделать жеребенка. Но нахуя это личности если от этого нет удовольствия? Дети требуют кучи ресурсов и времени и просто так, это проеб времени. Но существуют поощрения, которые дают впрыск гормонов и мотивируют продолжать что бы получить еще.
Аноним 11/04/16 Пнд 12:00:24 #76 №282016 
>>281939
Ну хуй знает, анон. Мне не совсем понятно зачем вообще такие вопросы задавать. Вот лично мне нравится поддерживать свою жизнедеятельность. Это интересно и приятно. Думаю, что 99,9% остального человечества руководствуются похожими принципами. Если ты не видишь дальнейшего смысла в поддержании популяции и всего вот этого вот - флаг тебе в руки, выпиливайся. Я пока побарахтаюсь, может, доживу до сингулярности, а там посмотрим.
Аноним 11/04/16 Пнд 12:00:40 #77 №282017 
Будут ли у Марсианина сиквелы с названиями Европеец про забытого космонавта на спутнике Юпитера Европа, Титанианин про забытого на Титане?
Аноним 11/04/16 Пнд 12:02:25 #78 №282018 
>>282017
Если ты их напишешь, то да, будут.
Аноним 11/04/16 Пнд 12:14:06 #79 №282024 
>>282006
>Братан, нож это хуевый пример. Опуская недостаточную проникающую способность
Братан, вот ты какраз плохой пример. Но я уверен, что если такой вопрос станет ребром, то есть шанс что ты просто начнешь чудеса творить этим ножом, начав в первую очередь использовать мозги.
Делай копье, делай ловушки, бегай быстрее, сделай уже лук.
Нож это инструмент, но к нему надо мозги. Бессмертым они тебя конечно не сделают, но шансы явно повысят.
Аноним 11/04/16 Пнд 13:22:26 #80 №282048 
14603701465900.jpg
>>282017
Европеец - хроники срачей политача 2012-2015, полное собрание сочинений.
Титанец одиночества - автобиография одного хиккана, или "как жить, когда всё быдло кругом уже переработано в метан и поэтому не получается ныть, что тебя никто не понимает (алсо нечем дышать)".
Марсианин: Приквел - отчёт о результатах миссий марсоходов NASA.
Лунянин - масштабное расследование лунных заговоров, полное собрание сочинений.
Уранец - практическое руководство по анальным игрищам и ядерной энергетике, исправленное и дополненное. С картинками.
Нептунец - краткое руководство по плаванию в газовых гигантах.
Плутонец - практическое пособие по улучшению качества фотографий путём сокращения расстояния между объектом съёмки и фотоаппаратом.
Мир курей - справочник по птицеводству в условиях сильной солнечной активности и глубокого вакуума.
Венерианец - ярко иллюстрированный справочник по ЗППП для студентов медицинских вузов.
Астеройд - справочник по грамматике для специалистов аэрокосмической отрасли, астрономов и астрофизиков.
Летающий елдак - книга по ракетостроению с мемасами))0) для учащихся средних классов общеобразовательных школ.
Водонагревающие устройства теплоснабжения в аэрокосмической отрасли - автобиография Джеффа Безоса.
Салюты, петарды и другая пиротехника для праздников - история компании Morton-Thiokol.
Посоны, извините, не удержался.
Аноним 11/04/16 Пнд 15:43:28 #81 №282124 
14603786084650.jpg
>>282048
>хроники срачей политача
Аноним 11/04/16 Пнд 16:34:37 #82 №282174 
>>281977
Да хуй знает. Почему мы вообще думаем, что физически мы с пришельцами в одной весовой категории? Может они карланы и весят меньше 10 кг? Таким пришельцам легче осваивать космос и создавать космические аппараты. Допустим там какие-нибудь паукообразные, или гоацины, или грызуны. Они могут в капсулу Союза штук 20 набиться. Конечно это накладывает определенные ограничения на их сооружения, так как ясно, что они не будут создавать капсулы для 20 изначально.
Может у них индивидуальные способности по выживанию падают гораздо более сильно при утере технологического уровня.
Так что тут параллели с обезьяной годны лишь потому что мы сами произошли от обезьяны, а вот от кого произошли пришельцы и как сильно они от предков отличаются - большой вопрос.
Аноним 11/04/16 Пнд 18:10:08 #83 №282243 
https://2ch.hk/d/res/340874.html

Пейсач, поскольку это более-менее общий тред, а пилить отдельный откровенно не хочется, оставлю тут. Всё же обсуждение раздела, как никак.
Аноним 11/04/16 Пнд 22:10:00 #84 №282364 
>>282174
бля ты рололо чтоли, или просто тупая обезьяна?
Метафора, це була метафора
Аноним 11/04/16 Пнд 22:43:00 #85 №282383 
Почему ракетная и реактивная техника имеет такой маленький ресурс? Вот авто ездят сотни тысяч километров с копеечными затратами на ремонт и обслуживание, сраные ступичные подшипники выдерживают ямы, камни и прочее говно. В ДВС надо только масло менять, да фильтры. А какой-нибудь реактивный самолет полетал-полетал и пиздец, надо две цены за него опять заплатить, чтобы он ещё смог немного полетать. Вроде бы кучу всего меняют там. С другой стороны пассажирские боинги всякие довольно долго летают и не дорого обслуживаются. Так в чем суть?
Аноним 11/04/16 Пнд 22:48:56 #86 №282385 
>>281953
Вообщем перед смертью не надышишся, во вторник уже пиздую на защиту, там и проверим все теории о сферах и прочем. Есть там кое-кто мозговитый. Отпишусь потом, если интересно.
Аноним 11/04/16 Пнд 22:54:08 #87 №282389 
>>282243
НЕБУДЕТ
Е
Б
У
Д
Е
Т
Аноним 11/04/16 Пнд 23:03:27 #88 №282392 
>>282385
ок, интересно
Аноним 11/04/16 Пнд 23:09:57 #89 №282396 
>>282383
Деталек много.
Дорогие высокопрочные материалы.
Да и если у авто колесо отвалиться высока вероятность отделаться травмами а если у самолёта крыло - пара сотен гарантированных трупаков
Аноним 11/04/16 Пнд 23:13:59 #90 №282400 
>>282396
А можно как-то сделать так, чтобы они почти не ломались? Или тогда это будет связано с понижением эффективности и увеличением затрат на топливо?
Аноним 11/04/16 Пнд 23:17:06 #91 №282404 
14604058262130.jpg
14604058262321.png
14604058262382.jpg
14604058262413.jpg
>>282383
Там совершенно ебовые нагрузки, понимаешь? ДВС не сравнить с ЖРД, где за секунду сгорает 2,5 тонн горючего и окислителя, которые за ту же секунду переходят от -180 до 800 градусов цельсия.
К тому же эта техника намного сложнее, с дохуярдом деталей из самых диковинных сплавов и композитов, потому-что предназначена для гораздо более сложных задач - катиться по земле это хуйня, а вот лететь над ней в управляемом полёте - уже кое что более сложное. А уж в космос хуярить так и вообще.
Аноним 11/04/16 Пнд 23:19:11 #92 №282405 
>>282400
Нет нельзя. И ещё есть человеческий фактор.
Аноним 11/04/16 Пнд 23:33:10 #93 №282411 
14604067906140.jpg
>>282383
>Почему ракетная и реактивная техника имеет такой маленький ресурс?
Нагрузки и их перепады близки к предельным возможностям известных материалов. Плюс большинство ракет одноразовые. Все это заставляет конструировать в расчете на строго ограниченное количество циклов работы.

> Вот авто ездят сотни тысяч километров с копеечными затратами на ремонт и обслуживание
Формулу-1 видел? Эти бешеные табуретки несколько десятков кругов не всегда могут проехать, а шины - никогда. Ради экономии спроектировано тютелька в тютельку, чтобы сломаться сразу после финиша. Вот так и тут.
Аноним 12/04/16 Втр 15:25:55 #94 №282657 
14604639558110.jpg
Почему не может быть Земли на орбите, вращающейся Юпитера так, чтобы на Земле была атмосфера, максимально идентичная земной?
Рассмотрим некую звезду, где есть Юпитер в зоне обитаемости, и вокруг него вращается его луна - Земля, по размерам, плотности и химсоставу (по крайней мере в толще пары километров поверхности) идентична нашей Земле.
Насколько близко/далеко такая Земля должна быть от своего Юпитера? Какого размера должен быть этот Юпитер? Как избежать того, чтобы такая Земля всегда одной стороной смотрела на свой Юпитер?
Аноним 12/04/16 Втр 15:47:43 #95 №282673 
>>282657
Нихуя себе вопросики
Аноним 12/04/16 Втр 16:52:56 #96 №282707 
>>282657
давай линку, кто сказал что не может
Аноним 12/04/16 Втр 17:17:06 #97 №282718 
Почему новостной тред не перекатывают?
Аноним 12/04/16 Втр 17:27:16 #98 №282721 
>>282718
Потому что ТЫ должен осознать себя как часть спейсача и сам начать перекатывать треды, которые тебе интересны. Выбери пикчу по теме, не обосрись с текстом в поле темы, по желанию запости в новом треде ссылку на предыдущий тред, а в том треде запости ссылку на новый тред. Ума много не надо, просто не ставь политоту в шапку нейтрально новостного треда.
Аноним 12/04/16 Втр 18:02:15 #99 №282741 
В вики прочитал, что взрыв сверхновой(которая сейчас Крабовидная туманность) наблюдали в 1054 году. НО БЛЖАД КАК, если сейчас мы ее видим собсна туманностью?
Ведь она на расстоянии 6500 световых лет от Земли.

Уже чую, что обосрался, но прошу разъяснить эту тему.

мимо(тупое)школоло
Аноним 12/04/16 Втр 18:11:56 #100 №282747 
>>282741
Взрыв произошёл 7462 года назад по «синхронизированному» времени. 6500 лет назад его последствия имели такую форму, как мы можем наблюдать сейчас. В действительности сейчас эта туманность уже гораздо больше, но актуальная информация достигнет Земли через 6500 лет.
Аноним 12/04/16 Втр 18:38:33 #101 №282757 
14604755139900.jpg
>>282392
КОНФА НАЧИНАЕТСЯ В 12
@
В 8 УТРА ЗВОНИТ ЗНАКОМЫЙ, КОТОРЫЙ ДОЛЖЕН ЕХАТЬ С ТОБОЙ
@
ГОВОРИТ, ЧТО У ЕГО РОБОТА-ПАУКА, КОТОРОГО ОН ПОКАЗЫВАЕТ НА КОНФЕ СГОРЕЛА ПЛАТА
@
ДОГОВАРИВАЙТЕСЬ ПИЗДОВАТЬ К КРУТОМУ ПРЕПОДУ-ТРУДОВИКУ В ЦЕНТРЕ ДЕТСКОГО ТВОРЧЕСТВА ДЛЯ ЗАМЕНЫ ДЕТАЛЕЙ
@
ПРИХОДИШЬ, МЕТРОБАБКА НА ВАХТЕ ОРЁТ БЕЗ БАХИЛ НЕЛЬЗЯ И ПОХЕР ЧТО ТЕБЕ НЕНАДОЛГО И ВСЁ РАВНО ИДЁШЬ В ЗАСРАННЫЙ ПОДВАЛ ГДЕ И ТАК ГРЯЗНО
@
ПОБОЯВШИСЬ СТАТЬ КОНТУЖЕННЫМ ОТ ЭТОГО ОРА ПОНИМАЕШЬ,ЧТО ПРИДЁТСЯ ИДТИ В БЛИЖАЙШУЮ АПТЕКУ
@
ВЫХОДИШЬ,ПЕРЕДУМЫВАЕШЬ
@
ИЩЕШЬ ПОДВАЛЬНОЕ ОКНО И БЬЁШЬСЯ ТУДА СЛОВНО ПОТЕРПЕВШИЙ
@
ОТКРЫЛИ, ЗАЛЕЗАЕШЬ
@
ЗНАКОМЫЙ ЕЩЁ НЕ ПРИШЁЛ
@
ЧЕРЕЗ НЕКОТОРОЕ ВРЕМЯ ГОРЕ ИЗОБРЕТАТЕЛЬ ПРИБЕГАЕТ И ГОВОРИТ, ЧТО КОНФА В 11
@
ВРЕМЯ 10
@
ЕЩЁ НУЖНО ДОБРАТЬСЯ ДО ДРУГОГО ГОРОДА
@
ПРЕПОД СОГЛАШАЕТСЯ ПОДВЕЗТИ ВАС НА СВОЁМ ДРАНДУЛЕТЕ И ЧИНИТЬ РОБОТА ПРЯМО НА КОНФЕ
@
СОБРАЛИ ИНВЕНТАРЬ
@
ГОТОВЫ ВЫДВИГАТЬСЯ
@
ЗНАКОМЫЙ РЕШАЕТ В ПОСЛЕДНИЙ РАЗ ПРОВЕРИТЬ НАЛИЧИЕ НА ФЛЕШКЕ ПРЕЗЕНТАЦИИ ХОТЯ УВЕРЕН ЧТО ОНА ТАМ ЕСТЬ
@
ЕЁ ТАМ НЕТ
@
ОБЫСКАВ ВСЕ КОМПЫ РЕШАЕТЕ ЕХАТЬ ТАК
@
ПРИЕЗЖАЕТЕ В ШАРАШКУ И ВИДИТЕ ОГРОМНЫЙ ЗАБИТЫЙ ЛЮДЬМИ ЗАЛ
@
ЕЩЁ НЕКОГДА НЕ ВЫСТУПАЛ ПРИ ТАКОМ КОЛИЧЕСТВЕ ЛЮДЕЙ
@
ПОХУЙ, БЕЖИТЕ НА ЭТАЖ ВЫШЕ УЖЕ К ДРУГОМУ ПРЕПОДУ, ВЕДЬ У НЕГО ДОЛЖЕН ХРАНИТСЯ ВТОРОЙ РОБОТ ЗНАКОМОГО КОТОРОГО ОН ДЕЛАЛ РАНЬШЕ И МОЖНО БУДЕТ СДЕЛАТЬ ЗАМЕНУ
@
ЕГО НЕ ПРИВЕЗЛИ
@
ЗАБЕГАЕТ ТЁТКА И ГОВОРИТ ВАС ВСЕ ЖДУТ
@
ИДЁШЬ ОДИН ЧТОБЫ ПОКАЗАТЬ ЧТО ВЫ ПРИЕХАЛИ, ЗНАКОМЫЙ ОСТАЁТСЯ НА ПОЧИНКУ СГОРЕВШЕГО РОБОТА
@
НАЗЫВАЮТ ЧЛЕНОВ ЖУРИ
@
СЛЫШИШЬ ЗНАКОМОЕ ИМЯ ЗНАТОКА СФЕР
@
ТАК КАК ШАРАШКА ИНФОРМАЦИОННОГО ПРОФИЛЯ ПОЛОВИНА ПРОЕКТОВ ЭТО ОЧЕНЬ "ПОЗНАВАТЕЛЬНЫЕ" САЙТЫ ОБ АСТРОНОМИИ СДЕЛАННЫЕ СТУДЕНТАМИ САМОЙ ЖЕ ШАРАШКИ
@
ВТОРАЯ ПОЛОВИНА ЭТО РОБОТЫ, ПРИ ЧЁМ ИНОГДА ОЧЕНЬ ГОДНЫЕ, НАПРИМЕР РУКА ПРОТЕЗ, КОТОРАЯ ДВИГАЕТ ПАЛЬЦАМИ
@
ТЕБЯ ВЫЗЫВАЮТ
@
РАССКАЗЫВАЕШЬ БЕЗ ТЕКСТА, ТАК КАК НЕОГРАНИЧЕН ПО ВРЕМЕНИ ИНОГДА ОЧЕНЬ ГРАМОТНО ИМПРОВИЗИРУЕШЬ
@
ВОЛНЕНИЯ НЕТ, СЛЫШИШЬ КАК ТЁТКА ИЗ ЖУРИ НА ПЕРВОМ РЯДУ ДАЖЕ ПРО СЕБЯ СКАЗАЛА "МОЛОДЕЦ", ЧУВСТВУЕШЬ СЕБЯ НЕВЬЕБЕННЫМ УЧЁНЫМ
@
ПОСЛЕ ВЫСТУПЛЕНИЯ ПОНИМАЕШЬ, ЧТО ЗНАТОК СФЕР КАК РАЗ ВОВРЕМЯ СЪЕБАЛ ПО АРХИВАЖНЫМ ДЕЛАМ
@
ТЕБЕ ГОВОРЯТ ПРИВЕСТИ ВТОРОГО УЧАСТНИКА И ОБЪЯВЛЯЮТ ПЕРЕРЫВ
@
ОКАЗЫВАЕТСЯ(!) ВТОРОГО РОБОТА ПРИВЕЗЛИ, И ПУТЁМ ОБЪЕДИНЕНИЯ ДВУХ ЭКЗЕМПЛЯРОВ УДАЛОСЬ ЧТО-ТО СДЕЛАТЬ
@
СЕРВОПРИВОД ПЕРДИТ ИСКРАМИ
@
ОРЁТЕ
@
С ДВУМЯ НОГАМИ ИЗ ШЕСТИ С ГОРЯ ПОПОЛАМ ХОТЬ ЧТО-ТО ПОКАЗАЛИ
@
В НАЧАЛЕ КОНФЫ ГОВОРИЛИ ЧТО НАШУ КОМПЕТЕНЦИЮ ПРОВЕРЯТ И БУДУТ ВОПРОСЫ
@
ИХ НЕ БЫЛО
@
ГОВОРИЛИ ЧТО ЖУРИ БУДУТ СОВЕЩАТЬСЯ И НАЗОВУТ ПОБЕДИТЕЛЕЙ
@
ОНИ НЕ СОВЕЩАЛИСЬ А ПРОСТО ПИЗДЕЛИ НЕ О ЧЁМ ДАЖЕ НЕ СКРЫВАЯ, ПОБЕДИТЕЛЕЙ НЕ ОБЪЯВЛЯЛИ И ТЫ ЗАМЕЧАЕШЬ КАК ГДЕ-ТО В СРАТОМ УГОЛКЕ ФОТКАЕТСЯ КОМПАШКА С ДИПЛОМАМИ, И ЭТО ТЕ СРАНЫЕ ШАРАШНИКИ С САЙТАМИ
@
ДАЖЕ РЕБЯТА С ПРОТЕЗОМ-РУКОЙ НИЧЕГО НЕ ПОЛУЧИЛИ
@
ПЕРЕД ВАМИ ВОЗНИКАЕТ ТЁТКА И ДАЁТ ГАЛИМЫЕ СВИДЕТЕЛЬСТВА, КОТОРЫЕ ДАЖЕ ПОДПИСЫВАТЬ САМИМ НАДО(КАРТИНОЧКА ПРИЛАГАЕТСЯ)
@
ПОНИМАЕШЬ ЧТО ЛУЧШЕ БЫ ЕЩЁ РАЗ ЕХАЛ В МОСКВУ
@
ЕДИНСТВЕННЫЙ ПЛЮС - ВЫСТУПЛЕНИЕ ПЕРЕД БОЛЬШОЙ АУДИТОРИЕЙ
@
СИДИШЬ НА ЛАВОЧКЕ В ПАРКЕ СМОТРЯ НА ЛИСТОЧЕК ДЛЯ ПОДТИРАНИЯ ЖОПЫ












Аноним 12/04/16 Втр 19:15:22 #102 №282794 
>>282757
Прохладная былина, космач.
Аноним 12/04/16 Втр 19:36:45 #103 №282807 
У Dragon есть сбрасываемый обтекатель?
Аноним 12/04/16 Втр 19:47:00 #104 №282813 
>>282794
Просто неприятная ситуация вышла, пригорало вот и написал. Столько проделать и ни хера не получить. Но в целом пофиг. Ведь скил выступления всё равно поднят. Такие вот дела.
Аноним 12/04/16 Втр 19:55:04 #105 №282824 
>>281752
забыл добавить относительно защиты от этой фигни,
но раз я нашел работника атомной промышленности на ютубке ему слово https://youtu.be/jDarcdMiIGs?t=352

Те в качестве материала для защиты, свинец не обязателен.
Это может быть и любой другой материал, грубо говоря той же массы.
Аноним 12/04/16 Втр 19:56:20 #106 №282826 
>>282757
состаришься будешь вспоминать, у меня тоже есть такая филькина грамота.
Аноним 12/04/16 Втр 20:11:21 #107 №282851 
>>282824
> Те в качестве материала для защиты, свинец не обязателен.
> Это может быть и любой другой материал, грубо говоря той же массы.

По результатам каких-то экспериментов на МКС: будет чучка люминия и овердохуя спец. пластика. Опционально(но сомнительно) лепестрический экран для электроники от вспышек.
Аноним 12/04/16 Втр 22:35:09 #108 №283090 
Если на орбите выстрелить из автомата в ретрогрейд, упадёт ли пуля на землю и сможет ли кого-нибудь убить?
Аноним 12/04/16 Втр 22:47:36 #109 №283108 
>>283090
На низкой упадет и без автомата, сама, рано или поздно. Но вообще для схода с НОО КА требуется дунуть на 100м/с приблизительно, у автомата в несколько раз больше. Нет, убить не сможет, расплавится задолго до земли. (если не свинцовая, то хз)
Аноним 12/04/16 Втр 22:48:11 #110 №283109 
>>282807
У самого Dragon нет. Есть у Falcon.
Аноним 12/04/16 Втр 23:00:08 #111 №283128 
14604912087440.gif
>>282657
Земная атмосфера имеет такой состав в основном от наличия на ней жизни. Проблема в том, что батя юпитер например вблизи и вдали фонит как росатый из-за своей могучей магнитосферы и радиационных поясов. Разве что где-нибудь на очень отдалённой орбите, как Феба вокруг Сатурна.

> Как избежать того, чтобы такая Земля всегда одной стороной смотрела на свой Юпитер?
На высокой орбите не попадет в приливный захват очедолго.
Аноним 12/04/16 Втр 23:13:43 #112 №283132 
>>283109
был же, колпачек на доке
на пуске было видно как его сбросили. была призентация драгона, там он открывался.
Аноним 13/04/16 Срд 00:07:15 #113 №283143 
>>282757
Ух ты, зема, с днем космонавтики, че. ТГПУ штоле?
Аноним 13/04/16 Срд 00:11:01 #114 №283145 
>>282757
У меня туфли такие же, на китайском рынке за 1000р. брал.
Аноним 13/04/16 Срд 02:05:56 #115 №283168 
14605023561370.jpg
>>281364
>Вот вам вопрос - допустим в точке лагранжа 1 находится корабль пришельцев. Как мы его обнаружим, если он прячется в лучах Солнца?
ОП-ПА! Попались, пришельцы ебаные! Еще и целую флотилию с собой приволокли
Аноним 13/04/16 Срд 02:17:53 #116 №283169 
>>283168
Хорошо шутишь, мне понравилось.
Аноним 13/04/16 Срд 02:46:00 #117 №283171 
14605047609750.png
>>283128
> приливный захват
А может ли планета дрейфовать на орбите за ближайшей точкой орбиты так, чтобы местный Юпитер в небе выглядел ни как Солнце или Луна, а как огромное массивное нечто, закрывающее хотя бы десятую часть обозримого неба?
Аноним 13/04/16 Срд 06:20:56 #118 №283184 
>>283171
Что ты подразумеваешь под десятой частью? 36 градусов угловым размером, или шоб прям площадь небесной полусферы на десятую долю закрывал?
Аноним 13/04/16 Срд 07:49:35 #119 №283189 
14605229753100.png
>>283132
Вижу пленку какую-то. Как её сбрасывают, интересно?
Аноним 13/04/16 Срд 09:08:42 #120 №283196 
14605277226000.jpg
>>283189
Я думаю её перед стартом снимают, как те чехлы с обтекателя протона.
Аноним 13/04/16 Срд 09:38:09 #121 №283205 
14605294895700.jpg
>>283196
Я понял, она остается, только окошки открываются. Наверно, их какой-то легко рвущуюся от ветра пленкой закрывают.
Аноним 13/04/16 Срд 11:32:34 #122 №283261 
>>283196
Вчера Криса нашего Хэчфилда читал, там упоминалось про пленку на Союзе, которая при возвращении в атмосферу обгорает. Может тут та же херня.
Аноним 13/04/16 Срд 12:17:52 #123 №283274 

>>282757
Проиграл. Годная кулстори, спасибо.
Аноним 13/04/16 Срд 12:25:48 #124 №283279 
>>283261
Теплоизоляция на твоих Союзах.
На Драконе, как и на Шатоле, заклеивают сопла, чтобы говна не натекло.
Кстати, почему только русские заворачивают корабли в ватники? Луноход тоже, кстати.
Аноним 13/04/16 Срд 12:25:57 #125 №283280 
14605395574800.jpg
>>282757
обучаемуся?
обучаейся?
обучаемуейся?

Вот уж действительно, листочек для подтирания жопы.
Аноним 13/04/16 Срд 12:27:45 #126 №283284 
>>282757
Как бывший участник подобной хуни аж на республиканском уровне, скажу: не будь ебланом, лучше полезное что сделай.
Аноним 13/04/16 Срд 13:33:48 #127 №283308 
https://meduza.io/news/2016/04/12/k-alfa-tsentavra-otpravyat-mini-korabli-na-neveroyatnoy-skorosti?utm_source=website&utm_medium=push&utm_campaign=breaking

Это вообще что и как?
Аноним 13/04/16 Срд 13:50:20 #128 №283314 
>>283308
https://2ch.hk/spc/res/282081.html
Аноним 13/04/16 Срд 16:09:02 #129 №283368 
Спейсач, смотри. Тяга реактивных двигателей возникает из-за того что при сгорании компонентов топлива резко растет их температура, а значит и давление. И под действием этого давления они с высокой скоростью выходят через сопло.

Почему рабочее тело нельзя тупо греть, например электричеством до значительно более высоких температур, тем самым увеличивая тягу при той же массе расходуемого топлива?

Или там все упирается в конструкцию сопла и температуры, которую оно может выдержать?
Аноним 13/04/16 Срд 16:42:31 #130 №283377 
>>283368
А электричество думаешь халявное? Материалы это тоже проблема, даже ядерный двигатель больше 900с импульса мы не осилим из-за отсутствия достаточно жаропрочных материалов.
Аноним 13/04/16 Срд 16:46:41 #131 №283382 
>>283377
Ну реактор ядерный да. Ну или копить энергию от солнечных батарей и разгоняться импульсами.

>даже ядерный двигатель больше 900с импульса мы не осилим из-за отсутствия достаточно жаропрочных материалов.

А у химических лучших сколько импульс?
Аноним 13/04/16 Срд 16:48:38 #132 №283384 
>>283377
>900с

и кстати почему единицу измерения импульса тут пишут "с", он же вроде как "кг*м/с" ?
Аноним 13/04/16 Срд 16:54:42 #133 №283388 
>>283368
Ты только что аркджеты. Которые не используются ИРЛ по причине того, что электричество негде достать. Используются только в гиперзвуковых аэродинамических трубах для продувки всякой хуиты. (см. NASA Arcjet Complex или ЦАГИшные трубы)
> Или там все упирается в конструкцию сопла и температуры, которую оно может выдержать?
Всё верно.

>Почему рабочее тело нельзя тупо греть, например электричеством до значительно более высоких температур
Можно, и делают. Только микроволновым радиоизлучением. Плазменные двигатели (вроде СПД или VASIMR) удерживают и ускоряют рабочее тело магнитным полем, т.к. это плазма.

Энивей, электричество на орбите достать трудно, все упирается в отвод тепла обычно. Плюс чтобы получить сколько-нибудь заметную тягу, нужно его ДОХУЯ.
Аноним 13/04/16 Срд 16:58:07 #134 №283393 
>>283382
Лучший на кислородно-водородной паре - вдвое меньше. Но с водородом полная пидорасня - очень быстро испаряется и плотность никакая, надо огромные баки городить. Реально в районе 300 импульс.
>>283368
Ты только что изобрел ионный, плазменный двигатели и прочую подобную поебень. Там правда не греют, а разгоняют, не совсем то.
>>283384
Либо м/с, либо с, и так и так можно.
>>283377
Есть разные ебанутые концепции возможности обхода предела жаропрочности, разные жидкосолевые ЯРД Омского КБ.
Аноним 13/04/16 Срд 16:58:55 #135 №283395 
>>283382
Батареи это масса и низкий кпд на данный момент. У классической химии 450-470с потолок(причем на уровне море такое хрен получишь). С трехкомпонентной экзотикой литий-фтор-водород 520.
Аноним 13/04/16 Срд 17:01:46 #136 №283397 
>>283393
Он не ионный или холловский изобрел, а Электротермические. У него и слова нет про ионизацию.
Аноним 13/04/16 Срд 17:13:07 #137 №283403 
>>283393
>Либо м/с, либо с, и так и так можно.

ну это тот же импульс который в физике?
1с значит мы разгоняем килограмм на 1м/с
или как?
Аноним 13/04/16 Срд 17:16:41 #138 №283406 
>>283403
Речь про https://ru.wikipedia.org/wiki/%D0%A3%D0%B4%D0%B5%D0%BB%D1%8C%D0%BD%D1%8B%D0%B9_%D0%B8%D0%BC%D0%BF%D1%83%D0%BB%D1%8C%D1%81
Аноним 13/04/16 Срд 17:55:15 #139 №283419 
>>283184
>шоб прям площадь небесной полусферы на десятую долю закрывал
Это
Аноним 13/04/16 Срд 19:01:16 #140 №283443 
14605632767730.jpg
>>282006
Двачую, бро. Что человек с ножом против гориллы? Да, нож острый и длинный, круче когтя тигра, но им-то еще нужно ударить. А ведь это не продолжение тела, не коготь, с которым человек родился, он просто не приспособлен пользоваться этим оружием. Ему нужно учиться, тренироваться, оттачивать умение и стратегию. А у гориллы все вшито в подкорку: хук справа - и пиздарики.

Поэтому я вообще считаю, что человечество обречено как вид. Инструменты эти - чахлые игрушки, ничто по сравнению с миллионами лет отточенными рефлексами. Копья эти, луки, ловушки - это все прекрасно, но ты ими скорей себя убьешь, пока научишься пользоваться. Только если с детства учить. Но дети у людей от рождения беспомощные орущие имбецилы, от которых один вред: демаскируют, срутся, взрослеют медленно. Короче, недоразумение сплошное, а не вид. Дайте им еще пару тысяч лет, они и вымрут нахуй.

Тогда заживем.
Аноним 13/04/16 Срд 19:07:57 #141 №283445 
>>283388
>нужно его ДОХУЯ.
Поправлю пропорционально квадрату улучшения импульса.
Те если импульс 400с и мы хотим 800с - надо будет вливать в 4 раза больше энергии.
Для импульса в 4000с надо уже в примерно в 100 раз больше энергии, чем можно выжать из кислород водородной пары.

>>283384
Это не импульс, это техническая единица-подход
Сколько секунд может создавать тягу в 1 килограмм силы, 1 килограмм топливной смеси. Забыл в какой системе, вообщем им там с этим удобней было.
Аноним 13/04/16 Срд 19:10:55 #142 №283448 
>>283143
Если ты про место выступления, то это был статистический технарь, туда много кто съехался. Несмотря на то, что он выглядит солидно по сравнению с другими учреждениями и организация была норм, выступление с самого начала не внушало доверия. Победителей такое ощущение выбрали ещё заранее. А то всё больше на показуху смахивало. Я не говорю давайте награду мне, там были работы, которые заслуживали награды, но хер там. Сравни руку-протез которая двигаться весьма свободно и сайт с парой таблиц и текстом.
Аноним 13/04/16 Срд 19:59:33 #143 №283459 
>>283448
фотки с рукой есть ?
Аноним 13/04/16 Срд 20:50:07 #144 №283476 
>>283445
>Поправлю пропорционально квадрату улучшения импульса.
>Те если импульс 400с и мы хотим 800с - надо будет вливать в 4 раза больше энергии.
>Для импульса в 4000с надо уже в примерно в 100 раз больше энергии, чем можно выжать из кислород водородной пары.

А ядерный реактор в корабле дорого? Ну или конденсаторы очень емкие, чтоб копить энергию на импульс.
Аноним 13/04/16 Срд 20:59:21 #145 №283482 
14605703614140.jpg
>>283476
Дорого, но дело не столько в этом, главное что тепло лишнее некуда девать - вакуум вокруг.
Аноним 13/04/16 Срд 21:15:09 #146 №283489 
Заполните пробелы.
Самые важные события:
50е - Спутник-1
60е - Гагарин, Аполлоны
70е - ???
80е - Шаттлы, Мир
90е - МКС
00е - ???
10е - Илон
Аноним 13/04/16 Срд 21:40:29 #147 №283496 
>>283489
семидесятые - долговременные орбитальные станции, GPS (70-80е)
девяностые - Мир, взрослый период автоматики - ДЗЗ, связь/телеком, орбитальные обсерватории, действительно сложные АМС
нулевые - МКС, действительно сложные АМС, расцвет частной космонавтики
Аноним 13/04/16 Срд 21:46:52 #148 №283499 
>>283496
У тебя требуемого филиала /b/ не получается: "действительно сложные АМС" всегда есть, будут, и получаются.
Аноним 13/04/16 Срд 22:45:22 #149 №283514 
>>283499
Какой филиал /б/? О чем ты?
Алсо, действительно сложные АМС - это марсоходы в первую очередь, т.к. автономные и универсальные. Частично Луноходы, но у тех автономность отсутствовала. Это качественный скачок по сравнению с традиционными межпланетными станциями. В 90-е я засунул по ошибке, карп в пакете вышел.
Аноним 13/04/16 Срд 22:56:34 #150 №283516 
>>283514
У марсоходов ты где там автономность нашел, болезный. Кусок кода на каждый пук ему присылают.
Аноним 13/04/16 Срд 23:00:27 #151 №283517 
>>283496
Орбитальные обсерватории запускали еще с 60-х. Это просто Хаббл винрарным вышел в итоге, не в последнюю очередь из-за своего диапазона.
Аноним 13/04/16 Срд 23:19:37 #152 №283528 
>>283476
импульс в данном случае не означает прерывистая работа.
Это удельный импульс, выше была ссылка, от анона который не умеет копировать ссылки с русской вики.

Ядерный реактор это неплохо, но это примерно тоже как спрашивать, а не плохо бы поставить движок от Як-40 на самолет братьев Райт. Да неплохо, но надо сначала подрасти до Як-40, ну или до того момента когда тобуретка на будет разваливаться хотябы просто от веса движка.

В практическом отношении это означает, что должна появится задача кажущаяся инвесторам привлекательной. Компоненты, в том или ином виде уже есть. короче их должна покусать бешенная муха https://geektimes.ru/post/227973/
https://geektimes.ru/post/253368/


Аноним 14/04/16 Чтв 00:43:10 #153 №283568 
>>280996 (OP)
Аноны, что вы думаете про аморфные полупроводники?
Схема такая: добываем на луне любой металл в чистом виде (там дохуя железа, например), добываем любой из следующих полупроводников http://pskgu.ru/ebooks/pawhohlov/pawhohlov_13_03.pdf
Наносим на подложку (хоть на сам грунт) слой металла, сверху чем-нибудь полируем его, на него напыляем полупроводник, на полупроводник напыляем тонкий полупрозрачный слой металла. Получаем солнечную батарею, которая в производстве достаточно элементарна, так как не требуется получать поликристаллы, не говоря уже о моно.
Соответственно, можно их делать и использовать прямо на месте.
Аноним 14/04/16 Чтв 00:50:07 #154 №283571 
>>283568
Можно. А зачем столько энергии?
Аноним 14/04/16 Чтв 01:00:15 #155 №283572 
>>283571
Запускать металлические глыбы по малозаселенным районам земляшки в борьбе за независимость.
Аноним 14/04/16 Чтв 01:26:24 #156 №283585 
>>283572
>Запускать металлические глыбы по малозаселенным районам земляшки в борьбе за независимость.
Эффекта можно добиться расходуя на 3 порядка джоулей меньше.
Аноним 14/04/16 Чтв 01:32:50 #157 №283589 
Суперсильный ядерный бабах по типу взрыва царь-бомбы может сдвинуть планетку с траектории?
Аноним 14/04/16 Чтв 01:43:43 #158 №283593 
>>283589
Зависит от размера и состава планетки и ЙОБистости заряда.
Аноним 14/04/16 Чтв 05:35:34 #159 №283611 
>>283589
урана на слойку столько не наберется.
Аноним 14/04/16 Чтв 07:07:25 #160 №283614 
>>283419
Тогда маловероятно. Слишком близко должен быть спутник.
Аноним 14/04/16 Чтв 10:08:59 #161 №283632 
14606177399590.jpg
Как считать максимальное время в тени для спутника на наклонной орбите, т.е. с учётом бета-угла? Вот да, учитывая время засветки на красной части орбиты в пикриле.
В тырнетах для околоэкваториальных только формулы, либо жопой ищу.
Аноним 14/04/16 Чтв 10:12:25 #162 №283633 
14606179457480.jpg
Как считать максимальное время в тени для спутника на наклонной орбите, т.е. с учётом бета-угла? Вот да, учитывая время засветки на красной части орбиты в пикриле.
В тырнетах для околоэкваториальных только формулы, либо жопой ищу.
Аноним 14/04/16 Чтв 10:14:03 #163 №283634 
>>283633
Сорян, интернет через чайник.
Аноним 14/04/16 Чтв 11:02:12 #164 №283641 
Топология ЧД. С этой воронкой ничего не понимаю. Есть более понятное? Как вообще 4 мерные объекты смотреть?
Аноним 14/04/16 Чтв 11:23:24 #165 №283645 
>>283641
Запустить 4 проекции на 2-мерную плоскость, по 2 на каждый глаз.
Аноним 14/04/16 Чтв 12:31:59 #166 №283660 
>>283568
аморфные полупроводники хорошо.

можешь также посмотреть https://en.wikipedia.org/wiki/Perovskite_solar_cell

обычные солнечные концентраторы тоже вариант - относительно низкотехнологично, и сердито.
Аноним 14/04/16 Чтв 12:56:03 #167 №283665 
>>283645
Хотет.
Аноним 14/04/16 Чтв 12:58:40 #168 №283667 
>>283633
Разве будет разница у отношении?
Аноним 14/04/16 Чтв 12:58:56 #169 №283669 
>>283667
В
фикс
Аноним 14/04/16 Чтв 13:44:41 #170 №283676 
>>283667
Считаем для наихудшего случая.
Если считаем для экваториальной - апогей полностью в тени.
Если орбита с неким наклонением, и апогей достаточно высоко - некая часть орбиты и сам апогей будут выходить за тень Земли.
В идеале, если солнечные батареи йоба-эффективные и мгоновенно заряжают аккумуляторы, а апогей на полшишечки выступает из тени - уже в 2 раза меньшая ёмкость нужна. Если орбита более вытянутая - уже не получится поделить на 2 и получить достаточно точный результат.
Аноним 14/04/16 Чтв 13:53:46 #171 №283680 
>>283676
Cобственно, картинка к первоначальному посту всё показывает, вот.
Аноним 14/04/16 Чтв 14:05:32 #172 №283683 
>>283676
Посаны, помогите. Ткнете носом в книжку/формулу - будет наконец-то полноценный калькулятор для KSP, дохуя точный и дрочибельный.
Аноним 14/04/16 Чтв 14:21:10 #173 №283690 
>>283667
Разница будет.
Планета движется вокруг солнца не в той плоскости, в которой спутник движется вокруг планеты. Т.е. наибольшее закрытие != случаю, когда солнце светит со стороны перигея, и бОльшая часть орбиты закрыта планетой.
Черт, ну прост же.
Аналогично, пытался найти формулу - не нашел. Но как-то считают же.
Аноним 14/04/16 Чтв 14:39:36 #174 №283702 
Сможет ли взрыв Бетельгейзе как то навредить Земляшке и её жителям?
Аноним 14/04/16 Чтв 17:22:07 #175 №283761 
>>283690
А нахуй тебе формула? Ты научный проект пишешь, или очередной твердый фантаст? У тебя есть участки, закрытые тенью планеты. Дуги окружности орбиты. Радиус орбиты и период вращения есть? Значит есть длина орбиты и усредненная скорость движения по ней. Высчитываешь длины соответствующих дуг так, на глаз, делишь на усредненную скорость - получаешь время за один период вращения. Ну а если это научная работа, то тут большой матан, орбита дюже эллиптическая, скорость в апоцентре и перицентре заметно различается, для более-менее точного ответа интегрировать нужно.
Аноним 14/04/16 Чтв 17:38:05 #176 №283766 
>>283761
Курсач, нет, вообще не космическая спецуха. Нашел способ примерно рассчитать на школьной математике, но там дохуя приблизительно. А надо бы нормально, ибо дохуя эксцентричные орбиты с приличным наклонением юзаются достаточно широко.
Аноним 14/04/16 Чтв 18:40:47 #177 №283792 
>>283766
Даже матан не проблема особо. Но найти норм формулы не могу, вот. Насовский документ отрыл, но це дохуя сложно и надо ковыряться долго, много необязательных для моих задач штук вычисляется.
Аноним 14/04/16 Чтв 20:27:50 #178 №283831 
>>283459
К сожалению нет
Аноним 14/04/16 Чтв 21:16:13 #179 №283853 
Я знаю вы тут все ученые и гипер начитанные, но я нет. Поясните мне, Юпитер образовался вместе с солнцем или как все остальные планеты?
Аноним 14/04/16 Чтв 22:17:43 #180 №283891 
Излучают ли нейтронные звёзды? Всегда считал что да, но один чувак из курса лекций говорит нет. Получается хуй отличишь чд от нз?
Аноним 14/04/16 Чтв 22:28:56 #181 №283905 
>>283891
Если бы они нихуя не излучали, их бы не было видно.

Во-первых, те НЗ что быстро вращаются, дают сильное радиоизлучение (радиопульсары), т.к. имеют охуительнейшее магнитное поле и вращаются десятки-сотни раз в секунду. По пульсарам можно часы сверять. или навигацию в космосе делать

Во-вторых, если НЗ вращается уже слишком медленно и имеет вокруг себя летающие говна, то они начинают на неё падать и излучать при этом рентген в ужасе.

Вот те, которые вращаются средне, и не имеют говен неподалеку, как раз с виду мало отличаются от обычных звезд, и ничего сильно особенного не излучают.
Аноним 14/04/16 Чтв 22:35:11 #182 №283910 
>>283690
если у тебя определенны параметры орбиты то особенной проблемы нет.
Эта штука сильно зависит от самой орбиты, не только от бета угла, по этому и рассматривают круговую, там параметр скорости и времени линейны.
Аноним 14/04/16 Чтв 22:36:04 #183 №283912 
>>283853
Как все. А почему про Сатурн не спрашиваешь?
Аноним 14/04/16 Чтв 22:36:54 #184 №283913 
>>283905
А в видимом спектре излучают, если на них ничего не падает?
Аноним 14/04/16 Чтв 22:40:33 #185 №283914 
>>283913
Излучают, потому что пиздец горячие, А остывают долго.
Аноним 14/04/16 Чтв 22:40:49 #186 №283915 
14606628490640.png
>>283913
Аноним 14/04/16 Чтв 22:43:02 #187 №283918 
>>283912
Да я вообще полный ноль, ничего не знаю, просто недавно смотрел передачу про Juno, который должен Юпитер изучать, так там и сказали, что Юпитер мог образоваться вместе с солнцем. Поищу тогда про Сатурн информацию, спасибо.
Аноним 14/04/16 Чтв 22:46:18 #188 №283922 
>>283918
Так и есть, он образовался вместе с солнцем, и собирал в себя всё говно, но основную порцию говна собрало солнце. И все остальные планеты тоже образовались вместе с солнцем, но им говна почти не досталось.
Аноним 14/04/16 Чтв 22:48:44 #189 №283925 
>>283922
Какое "говно"? Водород и гелий?
Аноним 14/04/16 Чтв 22:49:53 #190 №283927 
>>283925
Да, окружающее вещество, из которого образовалась солнечная система. А когда солнце зажглось, то уже не смогло продолжать собирать говно, и свет отнёс говно подальше от солнца.
Аноним 14/04/16 Чтв 22:54:08 #191 №283929 
>>283927
А теоретически получается Юпитер, Сатурн, Уран, Нептун могут нахер взорваться?
Аноним 14/04/16 Чтв 22:55:31 #192 №283931 
14606637314020.gif
14606637314041.webm
>>283913
Большая часть приходится на рентген и гамму, но некоторое количество излучают и в видимом. В основном это синий свет, который обычно испытывает красное смещение из-за того, что НЗ это релятивистский объект. В общем-то нейтронных звезд бывает дохуя разных, но типичная изолированная небольшая НЗ светит красным или в ИК.
Аноним 14/04/16 Чтв 22:56:14 #193 №283932 
>>283929
Не могут, у них ничтожно мало массы для начала термоядерных реакций.
Аноним 14/04/16 Чтв 22:58:07 #194 №283935 
>>283932
Вроде поехвашие американци запускали аппарат с таблетками прессованного урана, должно было хватить
Аноним 14/04/16 Чтв 22:58:35 #195 №283936 
>>283935
американцы*
Аноним 14/04/16 Чтв 23:15:59 #196 №283944 
>>283935
Нет, не должно было. При всем желании, хоть ядрену бомбу туда в него кидай, хоть 10.
Аноним 14/04/16 Чтв 23:36:45 #197 №283955 
>>283935
на два порядка не хватает массы.
то что пиздануло, это возможно, возможно даже своеобразная слойка получилась, до по поджига у юпитера просто не хватает критической массы.
Аноним 15/04/16 Птн 00:15:43 #198 №283965 
Что будет, если выебать черную дыру? Допустим есть одна сверхмассивная ЧД, как твоя мамка я нахожусь на самом горизонте и присовываю за горизонт свой хуй. Что с ним произойдет?
(Автор этого поста был забанен. Помянем.)
Аноним 15/04/16 Птн 00:17:30 #199 №283966 
>>283965
/spc образца 2016
Пиздец
Аноним 15/04/16 Птн 00:19:51 #200 №283967 
>>283965
Останешься без хуя
Аноним 15/04/16 Птн 05:36:56 #201 №284008 
Если бы Земля превратилась в неодимовый магнит, какой величины было бы магнинтое поле? И могло бы оно долго сохраняться, или полюсы начали бы притягиваться, мять и плющить планету, пока поле не ослабнет?
Аноним 15/04/16 Птн 08:59:44 #202 №284011 
>>283965
Хуй никогда не вернётся к тебе.
Аноним 15/04/16 Птн 09:57:48 #203 №284015 
Пейсач, есть какое-то научное название у функии эф от икс равно икс в степени "е"? Типа экспонента, но икс и е местами заменены. Или просто показательная функция без выебонов?
Аноним 15/04/16 Птн 10:06:27 #204 №284019 
>>284015
Показательная функция.
Аноним 15/04/16 Птн 11:02:58 #205 №284022 
>>283910
Окей. Имеем перицентр, апоцент, наклонение, радиус тела и всё такое. Максимальное закрытие в случае высокоэксцентричной наклоненной орбиты случается тогда, когда большая ось орбитального эллипса перпендикулярна вектору от звезды до планеты. Как вычислять?
Аноним 15/04/16 Птн 11:03:31 #206 №284024 
>>284022
проекция оси на плоскость, в которой планета и звезда, т.е.
Аноним 15/04/16 Птн 11:21:16 #207 №284026 
>>283702
Далековато. То есть три с половиной неудачника сдохнут от рака, немного повредится озоновый слой и сдохнут еще три с половиной австралийца, но на австралийцев похуй.
>>283929
Теоретически в самом центре Юпитера могут происходить термоядерные реакции раз в полгода. Но до самоподдерживаеющейся реакции (даже чтобы пережечь литий и подобный кал) ему не хватает массы в несколько десятков раз.

(Автор этого поста был предупрежден.)
Аноним 15/04/16 Птн 12:44:22 #208 №284036 
>>284026
>Теоретически в самом центре Юпитера могут происходить термоядерные реакции раз в полгода.

Ссылку на статью дайте на это. По мне так хуета эта с потолка взята.
Аноним 15/04/16 Птн 13:11:05 #209 №284038 
>>284026
А если представить умозрительно, что юпитер может зажечься. Нам это чем-то грозило бы? Он же далеко.
Аноним 15/04/16 Птн 13:39:42 #210 №284041 
>>284038
Артур Кларк писал об этом в одной из своих Одиссей. В целом, ничего особенного. Смена дня и ночи пошла бы по пизде, правда, и многие виды, зависимые от нее, последовали бы следом. Но это самое страшное.
Аноним 15/04/16 Птн 13:44:11 #211 №284042 
>>284041
>Смена дня и ночи пошла бы по пизде

почему? он же намного дальше солнца и был бы заметно тусклее
Аноним 15/04/16 Птн 13:54:41 #212 №284044 
>>284042
В любом случае, он был бы ярче полной луны, если верить книге.
Аноним 15/04/16 Птн 14:07:43 #213 №284047 
Что будет если поссать в космосе? Например сделать в скафандре дырку и через катетер поссать?
Аноним 15/04/16 Птн 14:20:31 #214 №284050 
>>284047
[Вопрос про говномочуеблю в космосе №872201995783]
Моча превратится в мелкие кристаллы льда.
>>284026
А этого за что предупредили?
Аноним 15/04/16 Птн 14:30:21 #215 №284052 
>>284022
Эмпирически удалось установить, что без матана можно получить искомое значение, умножив ODT орбиты Pe x Pe на 1+эксцентриситет Ар х Ре.
Ну, хоть как-то.
Аноним 15/04/16 Птн 14:47:57 #216 №284055 
>>284050
А вакуум писюну ничего не сделает?
Аноним 15/04/16 Птн 14:58:08 #217 №284058 
>>284050
У него был пик с голой теткой (ужас какой!)
Аноним 15/04/16 Птн 14:58:34 #218 №284059 
>>284055
Удлиннит на 5-7см.
Есть такая методика.
Аноним 15/04/16 Птн 15:21:28 #219 №284064 
14607228883400.jpg
Суп SПС!

Поясни пожалуйста за такой момент. Вот есть закон сохранения импульса.

Гляди, искусственные космические объекты улетают с поверхности земли. Значит ли это что какой нить космический корабль стартовав, меняет хоть на полшишечки орбиту земли.

И развивая мысль, если достаточно долго и в больших количествах запускать корабли (или мусор к примеру чтоб от него избавиться) с земляшки в дальний космос можно ли ее (земляшку) опрокинуть на солнце, допустим?

Ну и еще один вопрос в ту же корзинку. Я правильно понимаю что в открытом космосе, гипотетический космический корабль вынужден, в виду того же закона сохранения импульса, что то из себя выбрасывать (частицы сгоревшего топлива) что бы двигаться в нужном направлении? Как же тогда быть? получается скорость/дальность полета как хочешь ограничена от количеством того что можно сжигать, взятого на борт? И, скажем, даже если яйцеголовые ученые сварганят СуперЕмкийЭнерджайзер (с) с огромным запасом Квт/ч в пальчиковой батарейке, это все равно не поможет?
Аноним 15/04/16 Птн 15:27:47 #220 №284065 
>>284064
>Гляди, искусственные космические объекты улетают с поверхности земли. Значит ли это что какой нить космический корабль стартовав, меняет хоть на полшишечки орбиту земли.
Да


>И развивая мысль, если достаточно долго и в больших количествах запускать корабли (или мусор к примеру чтоб от него избавиться) с земляшки в дальний космос можно ли ее (земляшку) опрокинуть на солнце, допустим?

Во-первых надо в этом случае запускать все корабли строго в одном направлении относительно орбиты земли иначе они друг друга будут компенсировать. Во-вторых да, если пол-планеты пульнуть в каком-то направлении, другие пол-планеты полетят в обратном. И когда исследовательские станции совершают гравитационные маневры вокруг земли, они тоже "забирают" у нее часть орбитальной скорости.


>Ну и еще один вопрос в ту же корзинку. Я правильно понимаю что в открытом космосе, гипотетический космический корабль вынужден, в виду того же закона сохранения импульса, что то из себя выбрасывать (частицы сгоревшего топлива) что бы двигаться в нужном направлении? Как же тогда быть? получается скорость/дальность полета как хочешь ограничена от количеством того что можно сжигать, взятого на борт? И, скажем, даже если яйцеголовые ученые сварганят СуперЕмкийЭнерджайзер (с) с огромным запасом Квт/ч в пальчиковой батарейке, это все равно не поможет?

Ну если у нас есть супер-батарейка и супер движок, то теоретически мы сможем выбрасывать частицы со скоростью света и топлива тащить надо будет не так уж много даже для путешествий к соседним звездам. Скажем 2/3 массы корабля всего лишь.

Аноним 15/04/16 Птн 15:30:41 #221 №284066 
>>284055
Вы о чём-то кроме хуёв и мочи думаете вообще?
>>284058
Омерзительно. Я сюда на ракеты дрочить пришёл, а не на баб.
Аноним 15/04/16 Птн 15:39:59 #222 №284069 
14607239996900.png
>>284064
> можно ли ее (земляшку) опрокинуть на солнце, допустим?
Можно, но на это уйдет OCHE много энергии, времени и массы. Тебе понадобится изменить скорость земли где-то на ~800 км\c (пикрелейтед)
> И, скажем, даже если яйцеголовые ученые сварганят СуперЕмкийЭнерджайзер (с) с огромным запасом Квт/ч в пальчиковой батарейке, это все равно не поможет?
Если ученые сварганят мега-энерджайзер, то ты сможешь разогнаться где-то до ~10% скорости света. Собственно, говоря, этот энерджайзер уже сварганили, ядрен батон называется: https://en.wikipedia.org/wiki/Project_Orion_%28nuclear_propulsion%29

А ещё можно зафигачить МЕГА-ЛАЗОР, посветить им на МЕГА-ЗЕРКАЛО, и за счет давления света разогнать это зеркало и все что к нему прикручено до чуть ли не 99% скорости света. Правда опять-таки, у тебя уйдет на это просто астрономическое кол-во времени и энергии.
Аноним 15/04/16 Птн 15:43:17 #223 №284070 
>>284069
>Можно, но на это уйдет OCHE много энергии, времени и массы. Тебе понадобится изменить скорость земли где-то на ~800 км\c (пикрелейтед)

Откуда 800 то? скорость движения земли по орбите солнца - примерно 30км/c. Если ее опустить до 0, то мы на солнце и упадем.
Аноним 15/04/16 Птн 15:58:24 #224 №284072 
>>284036
Да они где угодно могут происходить - где подходящие элементы есть. Ясен хуй, среди частиц есть те которые движутся быстрее и те которые движутся медленнее (хвосты максвелловского распределения). Рано или поздно найдутся имеющие достаточную скорость для преодоления кулоновского барьера. Другое дело, в баллоне со сжатым водородом ждать придется подольше, чем в Юпитере - где условия относительно близки к требуемым.

Мочер, ты совсем ебанулся? Баб голых постить нельзя, не тематика? А порашный постинг при каждом пуске тебе тематика? На тебе одетую бабу.
(Автор этого поста был предупрежден.)
Аноним 15/04/16 Птн 16:50:53 #225 №284080 
>>284070
Ошибся. Бывает.
Аноним 15/04/16 Птн 17:01:12 #226 №284082 
>В ходе проверки внешней обшивки МКС, на соскобах с поверхности корпуса и иллюминаторов были обнаружены следы жизнедеятельности морского планктона.
Это как?!
Аноним 15/04/16 Птн 17:02:56 #227 №284083 
14607289764590.jpg
Если бы сегодня пилили аналог ракет на луну, были бы они меньше?
Аноним 15/04/16 Птн 17:14:23 #228 №284089 
>>284083
Нынче бы на Луну летели по частям, ящитаю. Зачем строить огромный дрын без задач, если можно по частям?
Аноним 15/04/16 Птн 17:17:28 #229 №284090 
>>284082
Ну вот так. Живучий сука, как оказалось.
Аноним 15/04/16 Птн 17:21:07 #230 №284092 
>>284072
То есть, как я и подозревал - хуйня с потолка.
Аноним 15/04/16 Птн 17:23:29 #231 №284094 
14607302095830.jpg
>>284083
Почему мунлендер выглядит как сарай? При взгляде на него он выглядит, словно из картона.
Аноним 15/04/16 Птн 17:29:33 #232 №284097 
>>284094
По той же причине, по какой оптимизированный по массе под конкретную задачу корабль в ксп выглядит примерно так же и ангарные космонавты кричат "НИКРАСИВА"
Аноним 15/04/16 Птн 17:32:30 #233 №284098 
>>284097
Меня смущают сами стенки, они на фото слегка погнуты, лол.
Кстати, если фото стыковочного узла в момент, когда экипаж переходил с лунного модуля в командный?
Аноним 15/04/16 Птн 17:36:54 #234 №284100 
>>284069
>А ещё можно зафигачить МЕГА-ЛАЗОР, посветить им на МЕГА-ЗЕРКАЛО,

У стругацких в "стране багровых тучь", (давно читал поправьте если ошибаюсь), некий корабль летел к венере светя сам на себя как раз таки еба лазером. В смонтированное на корабле же зеркало в хвостовой части.

Правильно я понял? Лазер может находится на корабле? с земли в корабль светить не надо?


Аноним 15/04/16 Птн 17:37:25 #235 №284101 
14607310450790.jpg
14607310450811.jpg
14607310450832.jpg
14607310450853.jpg
>>284094
А он такой и был фактически, чуть ли не ногой можно было проломить. Всё из-за экономии. Даже вариант с нижними обзорными иллюминаторами завернули - слишком тяжелый был. В результате пришлось городить хитровыебанный цирковой номер на траектории столкновения с Луной, чтобы хоть примерно рассмотреть место посадки. http://spaceref.com/missions-and-programs/nasa/apollo/apollo-lunar-landing-mission-symposium/apollo-lunar-module-landing-strategy.html - платиновая уже ссылка

Ну и ЭВТИ любую суперсовременную ёбу делает похожей на пакет с мусором. Там же специально заворачивают не гладко, а чтобы чутка топорщилось.
Аноним 15/04/16 Птн 17:39:39 #236 №284103 
>>284083
Нет. Пилотируемый корабль сколько весил, столько и весит. Даже больше надо было бы, или мультипуск, если хоть что-то делать помимо флаговтыка.
Аноним 15/04/16 Птн 19:22:08 #237 №284130 
Как должна выглядеть схема стартового комплекса для аэрокосмической системы с воздушным стартом? Какие наземные сооружения необходимы?
Аноним 15/04/16 Птн 19:36:00 #238 №284137 
Можно ли сделать на основе мха и такой-то матери замкнутую экосистему с кислородом и прочим, способную человека со всяким перерарабывающим оборудованием обеспечить необходимым для жизни хотя бы лет на 100?
Такая ультимативная сычевальня? Видел концепт "вечного террариума" из мха, пишут, лет 15-20 без кислорода снаружи живет. И каких размеров будет такая сычевальня? Можно ли ее запихать под землю вместо космоса на случай ВСЁ планеты? А обеспечить полноценную жизнь на Луне? В целом, не понимаю, почему нельзя с помощью науки сделать то же, что наша планета, она тоже суть замкнутая экосистема плюс солнце, которое можно заменить энергией атома или еще чем.
Аноним 15/04/16 Птн 19:49:39 #239 №284141 
>>284130
Аэродром, здоровый МИК, чтобы можно было закатить туда самолет и посадить ему на хребет ракету. Баки с горючим. Наверное, все.
Аноним 15/04/16 Птн 19:50:34 #240 №284142 
>>284141
МИК разве не относится к техническому комплексу?
Аноним 15/04/16 Птн 19:53:06 #241 №284143 
>>284137
Делали что-то такое. Вроде бы, жить можно было, но не очень весело. Почитай.
https://ru.wikipedia.org/wiki/Биосфера-2
Аноним 15/04/16 Птн 19:54:01 #242 №284144 
>>284137
Потому что солнце и земля это необслуживаемые системы. Они не ломаются. Мы так делать не умеем.
Аноним 15/04/16 Птн 20:06:59 #243 №284149 
>>284142
Ну тогда просто аэродром.
Аноним 15/04/16 Птн 21:57:09 #244 №284176 
>>284098
Они не погнуты. Алюминиевый силовой набор покрыт бутербродом из кучи всякой хуйни. Гугли, блядь, заебали, в каждой статье пидоры-конспирологи пиздят за картонный LEM, и там же всё опровергается. А ещё заебали мифы про пробитие корпуса отвёрткой. Сука. Ааар!
Аноним 15/04/16 Птн 22:49:28 #245 №284190 
>>280996 (OP)
4 пик. Это стеб? Ведь понятно, что такая вакуумная поебень не сработает.
Аноним 15/04/16 Птн 23:08:53 #246 №284195 
14607509331780.jpg
>>284190
Нет, это серьёзнейшая научная схема, на создание которой положили несколько лет кропотливой работы лучшие умы человечества. Несколько месяцев ушло только на создание красной пунктирной линии, а пока учёные занимались теоретической проработкой схемы, несколько лучших дизайнеров мира стирали руки в кровь, рисуя стартовую площадку в мельчайших деталях. Такие малозначительные вещи, как поиск лекарства от рака и расшифровка человеческого генома были отложены на годы, потому-что все суперкомпьютеры Земли были заняты рассчётом траектории на четвёртой пикче с точностью до нанометра. Несколько спутников России и США провели подробное радиосканирование Эвереста, пока команды геологов занимались подробными исследованиями in situ. Методом селекции был выведен новый вид жидкого вакуума, что будет залит в трубу, проектирование которой уже почти завершено, и Китай собирается приступать к невиданному доселе объёму производства суперчугуния для неё, в то время как самые крупные наркокартели Бразилии и Мексики начали финансировать постройку второй ступени будущей ракеты Союз-Фалькон-V Ариан Хэви, которая будет работать на чистом сжиженном кокаине.
А тут приходит какой-то фуфел и говорит:
>Это стеб?
Аноним 15/04/16 Птн 23:10:32 #247 №284196 
>>284195
Не могу прочитать твою пасту. Уставился на "сжиженный кокаин" - хотет.
Аноним 15/04/16 Птн 23:50:25 #248 №284216 
>>284082
Восходящие потоки воздуха занесли. Они постоянно туда тваое говно несут.
Аноним 15/04/16 Птн 23:51:38 #249 №284217 
14607534981830.jpg
>>284195
>А тут приходит какой-то фуфел
Этого пидора в Химках видал - деревянными членами торгует!
Аноним 16/04/16 Суб 00:01:16 #250 №284219 
>>284052
Эй, йоба, что, ни одного студента с какой-нибудь астронавигации? Я хуй знает, к кому обращаться с такими запросами. Нету формул, и хоть в розетку ебись. Матановый препод вообще посмотрел как на поехавшего, "какие орбиты, какие тени, бери тройной интеграл от хуя и не выёбывайся, сука!"
Не хочу приблизительно, хочу красиво.
Посоны, ткнёте в матан - будет охуенный калькулятор для KSP, честно.
Аноним 16/04/16 Суб 00:46:31 #251 №284221 
>>284219
Да там же все довольно просто и умозрительно.
Какими данными ты располагаешь?
У тебя есть для нужного тебе момент угол наклонения оси звезда-планета к орбите спутника? Если есть все очень просто вывести.

Если нет - давай данные, которыми располагаешь
Аноним 16/04/16 Суб 00:57:43 #252 №284227 
>>284022
>Максимальное закрытие в случае высокоэксцентричной наклоненной орбиты случается тогда, когда большая ось орбитального эллипса перпендикулярна вектору от звезды до планеты. Как вычислять?

Ты не напутал? Если большая полуось перпендикулярна вектору от звезды то планеты, то это 2 случая:

1) Вся плоскость орбиты перпендикулярна этому вектору (грубо говоря полярная орбита, обращенная к солнцу) тогда закрытие у тебя 0

2) Когда плоскость орбиты лежит в плоскости орбиты планеты вокруг солнца. Но тогда в тени будет лежать та часть орбиты, которая ближе к перицентру, где скорость большая, и аппарат пролетит тень быстро.

Наибольшее закрытие по времени будет как раз тогда, когда большая полуось параллельна вектору от солнца до планеты и перицентр обращен к звезде. Тогда самый медленный участок орбиты будет аккурат в тени.
Аноним 16/04/16 Суб 00:59:36 #253 №284228 
>>284195
> Союз-Фалькон-V Ариан Хэви
Я попытался представить себе эту йобу из йоб. Не получилось, но кончил два раза.
Аноним 16/04/16 Суб 00:59:55 #254 №284229 
14607575959290.jpg
>>284221
Кароч.
Могу какие угодно параметры орбиты достать.
Проблема именно в вычислении максимального времени закрытия для эксцентричных и одновременно наклоненных орбит. Для экваториальных эксцентричных есть формула, работает, круто. Для всяких "Молний" - нихуя.
См. пикрил, например.

>>284227
А теперь подумай, что будет, если орбита дохуя эксцентрична и наклонение скажем 45 градусов. Вычислить для орбиты, лежащей в одной (или почти одной) плоскости с планетой и звездой - просто. Если наклонение большое - уже именно при звезде "сбоку" получается максимальное время затенения.
Аноним 16/04/16 Суб 01:02:08 #255 №284230 
>>284229
Посоны, сорян, конечно, но я только документ NASA нашел, где столько матана, что я охуел, и вообще дифуры во все поля и нет очевидного способа. А сам я не то чтобы математик, чтобы как-то выёбываться и извращаться.
Прошу прощения, если речь сбивчива: наебенился фенибутом, и залил всё вискарем.
Аноним 16/04/16 Суб 01:03:23 #256 №284231 
>>284229
Добавлю, что именно из-за этого, видимо, все калькуляторы для той же KSP считают именно для орбит, лежащих в плоскости экватора. Видимо, наклонение дичайше усложняет задачу.
Аноним 16/04/16 Суб 01:08:06 #257 №284233 
Ананасы, поясните за skylon. Выгодно ли? Взлетит ли?
Аноним 16/04/16 Суб 01:09:56 #258 №284236 
14607581961760.jpg
>>284229
>Если наклонение большое - уже именно при звезде "сбоку" получается максимальное время затенения.

Вот так ты имеешь ввиду? Это быстрый участок орбиты. Максимальное затенение будет, если солнце будет "снизу"
Аноним 16/04/16 Суб 01:10:04 #259 №284237 
>>284233
Не выгодно. Не взлетит.
Аноним 16/04/16 Суб 01:11:24 #260 №284238 
>>284236
Проблема только в том, что оно никогда не будет "снизу". Т.е. максимальное затенение будет именно при звезде "сбоку". Как рассчитать - хуй знает.
Аноним 16/04/16 Суб 01:12:17 #261 №284239 
14607583378170.gif
>>284231
>Добавлю, что именно из-за этого, видимо, все калькуляторы для той же KSP считают именно для орбит, лежащих в плоскости экватора. Видимо, наклонение дичайше усложняет задачу.

Да ничего не усложняет.
Тебе надо

1) найти углы орбиты между которыми у тебя тень.
2) применить уравнение Кепплера. Никакого жуткого матана в нем нет.


Аноним 16/04/16 Суб 01:14:56 #262 №284240 
14607584966040.jpg
>>284238
Ты что-то не то пишешь.
вот при такой орбите условно у тебя будет самое большое затенение
Аноним 16/04/16 Суб 01:16:26 #263 №284241 
>>284239
Поясни за "углы орбиты, между которыми у тебя тень" , не совсем понял. Засветка же "боковая" при большом наклонении, вообще не ебу, как это делать.

>>284240
Эм. Так. Ок. Ты всё правильно написал, только орбита с неким наклонением никогда не будет в таком положении. Она никогда не будет в одной плоскости с планетой и звездой.
Аноним 16/04/16 Суб 01:26:07 #264 №284242 
14607591677340.jpg
>>284241
Да тебе похуй на боковая или не боковая и на наклонения похуй.

Тень от планеты это условно "труба" в пространстве. Она пересекает твою орбиту в двух точках "вход в тень" и "выход из тени". Тебе нужны углы между большой полуосью эллипса и этими точками. А дальше уравнение Кепплера даст тебе угол на условной круглой орбите с известным радиусом который соответствует "углу тени".

Для круговой орбиты с известным радиусом мы легко посчитаем время за которое мы пройдем этот угол.
Аноним 16/04/16 Суб 01:31:21 #265 №284243 
14607594818290.png
>>284241
В общем вот наихудший случай для вытянутой наклоненной орбиты, как-то так. Камера от звезды, отмечен самый длительный тёмный участок орбиты.

>>284242
>круговой
Кому нужны круговые орбиты?..

Я, конечно, объебался, но нихуя не понял, как Вы предлагаете рассчитывать этот случай. На всякий случай, сегодня больше постить не буду.
Аноним 16/04/16 Суб 01:36:49 #266 №284246 
>>284243
>Я, конечно, объебался, но нихуя не понял, как Вы предлагаете рассчитывать этот случай. На всякий случай, сегодня больше постить не буду.

А зачем я тогда буду распинаться, если ты больше постить не будешь?
Аноним 16/04/16 Суб 01:37:52 #267 №284247 
>>284246
Окей, это было на случай, если я не понимаю, о чем ты из-за веществ.

Блин, круговая - всё как бы ясно.
Эксцентричная - низхуя не ясно, уж извините.
Аноним 16/04/16 Суб 01:38:59 #268 №284248 
>>284243
>Кому нужны круговые орбиты?..

Епта. Про круговую орбиту мы все знаем, так?
Уравнение кепплера - это связь между круговой орбитой и эллиптической. Оно "углы" на эллиптической преобразует в углы для круговой, для которой и время и скорость считать легко.

Я же постил гифку выше.
Аноним 16/04/16 Суб 01:40:59 #269 №284249 
>>284248
Ладно, извини, блядские таблетки действительно повышают доступность алкоголя, и сильно тормозят к тому же. Перечитаю, как приду в себя.
Аноним 16/04/16 Суб 01:50:02 #270 №284250 
>>284100
У Стругацких был термоядерный реактор и зеркало из другого набора лептонов.

Когда лазер на корабле, зеркало не нужно. Разворачиваешь его как сопло и светишь.
Аноним 16/04/16 Суб 01:54:23 #271 №284252 
>>284250
Эффективно пиздец.
Аноним 16/04/16 Суб 02:38:47 #272 №284259 
1. В космосе когда нибудь проводились эксперименты с беременностью хотя бы животных? Что родится при этом например у кошки?
2. Что будет испытывать организм в невесомости, если долго не ссать?
Аноним 16/04/16 Суб 02:47:47 #273 №284261 
>>284259
Птичек вылупляли вроде каких-то, не помню чем окончилось.
Аноним 16/04/16 Суб 02:54:53 #274 №284264 
>>284259
>2
Обоссытся?..
Аноним 16/04/16 Суб 03:07:44 #275 №284270 
>>284259
Читал что эмбрион не сможет нормально сформироваться
Аноним 16/04/16 Суб 03:13:51 #276 №284273 
>>284249
ну прийдешь в себя завтра - пиши. буду у компа расскажу как считать.
там все просто
Аноним 16/04/16 Суб 15:20:37 #277 №284369 
14608092371000.png
Как В кубомегопарсеке, стабильно и равномерно поддерживать 100 кПа атмосферного вздуха?
Аноним 16/04/16 Суб 15:31:25 #278 №284370 
>>284369
Никак.
Аноним 16/04/16 Суб 15:32:02 #279 №284371 
Сколько весила земля на заре своего существования Когда только только луна откололась по сравнению с сейчас?
Аноним 16/04/16 Суб 15:38:58 #280 №284374 
14608103389230.png
Как насчёт такой схемы. Скукожит ли такой куб, где слоями воздух и каркас?
Аноним 16/04/16 Суб 16:46:36 #281 №284384 
>>284374
Скукожит. И просто воздух тоже скукожит. И будет новая звезда. Аминь.
Аноним 16/04/16 Суб 19:46:36 #282 №284396 
>>284008
www.nifs.ac.jp/report/NIFS-886.pdf
Аноним 16/04/16 Суб 19:55:47 #283 №284398 
>>284064
>скорость полета ограничена количеством того что можно выбросить за борт?
https://ru.wikipedia.org/wiki/Циолковский,_Константин_Эдуардович
(Автор этого поста был предупрежден.)
Аноним 16/04/16 Суб 20:15:20 #284 №284401 
Новых модераторов же набрали лол
Аноним 16/04/16 Суб 20:51:25 #285 №284421 
>>284401
>>284398
Заебись, читал /d/ и крестился слава богу у нас ретивых нету и вполне умеренно все
ан нет, таки есть, пошел в д
Аноним 16/04/16 Суб 23:37:42 #286 №284476 
>>284398
Простите, а что не так с этим постом?
Аноним 16/04/16 Суб 23:52:45 #287 №284479 
14608399652020.jpg
14608399652621.png
Я чейто нихуя понять не могу котаны, звезд мы не видим из за хуевых камер, если только с выдержкой снимать, но обратите внимание на видяшки ночной земли с МКС, огни ночных городов камера тоже захватить не может? ВСЕ сука видно с ночными городами на выдержке сняты, таймлапсом или еще какой-нибудь хуитой, в реальном времени всегда вместо огней и сияний - чернота, пустота и тленность. Как видят все космонавты - как с первого или со второго пика?
Аноним 17/04/16 Вск 00:32:19 #288 №284489 
Является ли принцип надевания скафандра Сокол одной из причин, из за которой в нем не получится погулять по открытому космосу?
Аноним 17/04/16 Вск 00:46:42 #289 №284491 
>>284479
Космонавты видят чуть больше чем камера. Иногда и яркие звезды на солнечной стороне. Динамический диапазон глаза - до 25 ф-стопов, камер - до 15, на практике меньше. Фото на МКС снимают топовыми никонами, у них хороший ДД, но все равно до глаза не дотягивает. Видео не знаю чем.
Аноним 17/04/16 Вск 02:13:56 #290 №284500 
>>284476
фотка бабы была без пизды и сисек, 0 порн контент, слегка помятая правда. пацаны тут годами баб не видят
затрет наверно
Аноним 17/04/16 Вск 02:18:13 #291 №284502 
14608486932670.png
>>284476
картинку забыл, там другая была
Аноним 17/04/16 Вск 04:36:33 #292 №284511 
>>284384
Но будет, не звезда а звездообразный объект.
Аноним 17/04/16 Вск 04:39:17 #293 №284512 
Где-то слышал предел на поглощение ЧД материи. Можно подробнее?
Аноним 17/04/16 Вск 06:00:51 #294 №284514 
>>284512
Теоретически предела вроде как нет. Другое дело, что объекты массой больше 50 млрд. M не наблюдаются. Нетуть таких, родной.
Аноним 17/04/16 Вск 06:18:50 #295 №284515 
>>284384
Какая нахуй звезда? Тут материи на десяток галактик хватит, если не больше.
Аноним 17/04/16 Вск 08:28:02 #296 №284518 
>>284515
Перед тем как как всё схлопнется в йоба-массивную ЧД будет несколько сверхъебучих гигантов. В чем проблема? Да и плотность слишком большая чтобы галактикой стало.
Аноним 17/04/16 Вск 08:44:22 #297 №284519 
Что будет с белым карликом после полного остывания? Будет некая планетоподобная штука с охуительной массой? А из чего будет состоять? Допустим, это остаток звезды, близкой или аналогичной по параметрам к Солнцу.
Вселенная Артур 17/04/16 Вск 10:04:01 #298 №284524 
14608766415810.jpg
Добрый вечер, народ.
Я не могу вам передать, насколько меня привлекает космос. Я считаю, что понимание всего космоса и его появления лежит далеко за рамками нашего понимания "нормально". У меня есть еще вопрос: вот выше нашей планеты есть космос, а что за гранью космоса? Где его начало или конец? Ведь как известно он ( космос ) состоит из материи, а у каждой материи есть начало и конец.
Есть еще немного странный вопрос, возможно вы его не поймете но я все же попробую подать свою мысль.
Вот есть наше время, где ты, как ты сам считаешь, жив. Но рано или поздно ты умрешь и твой разум умрет вместе с тобой ( ты не будешь помнить эту жизнь ) но тем не менее ты можешь сейчас мыслить и делать разные выборы. Понимаешь меня? Черт, столько мысль на эту тему в голове, аж трясет.
Анон, что ты сам думаешь на эту тему? Мне будет очень приятно, если ты выскажешься, как я.
Аноним 17/04/16 Вск 10:13:29 #299 №284525 
>>284519
>Что будет с белым карликом после полного остывания?
https://ru.wikipedia.org/wiki/Чёрный_карлик
Аноним 17/04/16 Вск 10:15:52 #300 №284526 
>>284518
Лол, нет.
Аноним 17/04/16 Вск 10:43:30 #301 №284528 
>>284064
Хватит получать глупые ответы друг. Даже если пустить в одну точку Земли миллиард атомных бомб, орбита едва ли изменится на пол шишечки. А уж говорить о каком либо масштабном влиянии человеческих кораблей на такое массивное тело как Земля просто абсурдно
Аноним 17/04/16 Вск 12:15:55 #302 №284538 
>>284524
Земля это тоже космос.
Всё остальное из вопроса к тематике не относится.
Аноним 17/04/16 Вск 13:07:01 #303 №284546 
Когда мы будем путешествовать во времени в прошлое?
Аноним 17/04/16 Вск 13:11:56 #304 №284548 
14608879161980.jpg
>>284546
>Когда мы будем путешествовать во времени в прошлое?
Никогда, товарищ дух: это армия.
Аноним 17/04/16 Вск 14:30:19 #305 №284572 
Понять не могу, это>>284524 это эзотерика или наркомания?
Аноним 17/04/16 Вск 14:35:23 #306 №284577 
>>284572
Это псевдофилософия и конечно же наркомания.
Аноним 17/04/16 Вск 14:53:15 #307 №284581 
>>284572
взят обычный вопрос пятилетнего пацана, замешан приемами софистики и теологии в некоторую форму, которая в конечном итоге подходит для среднетонкого троллинга, на мой взгляд.
Реагировать, имеет смысл, только если тебе нравится форма.
Сам по себе вопрос валиден и обсуждался в прошлом или позапрошлом треде, есть ли конец у universum`а.
Это относительно части номер 1, часть номер два приклепана паравозом. Но тоже в принципе может быть валидным отражением состояния анона, не связанной при этом с расстройствами.(но может и не быть).

Пикча стебная, это вот точно могу сказать, просто на злобу дня, не в бровь, а в глаз, и я не об отдельных элементах, а о картине в целом.
Аноним 17/04/16 Вск 15:32:03 #308 №284598 
14608963238390.jpg
>>284581
>Сам по себе вопрос валиден
В текущих научных реалиях - не очень. Любые вмеметагалактические гипотезы (например, о топологии Вселенной) очень спорны и в основном ненаучны. Так что это скорее вопрос методологии науки, можно ли вообще говорить об этой самой топологии юниверсума.
Аноним 17/04/16 Вск 15:46:54 #309 №284614 
>>284598
Ну вычурные матетамтические абстракции, на момент создания не имевшие физического смысла, потом удивительным образом находили свое отражение в реальности.
Толи мы предсказали эту реальность мат аппаратом, толи нашли применение этой конструкции.
Комплексные числа, тензоры, фазовая скорость, в электродинамике чтото такое было, сто и тд.
Вопрос в любом случае проходит какие то свои этапы эволюции: осознание вопроса, поиск формулировки, обоснование, поиск ответа.
И то что кажется что в нем нет смысла, не всегда хороший повод его не рассматривать.
Это как наса, практического смысла как бы ноль, но сам поиск приносит результаты.
Но я пожалуй все сказал, по поэтому вопросу, я не большой знаток сабжа, не интересует, так как практического смысла я не вижу(но если кто то кто будет на него отвечать случайно изобретет антиграв, мне будет не в чем себя корить).
Аноним 17/04/16 Вск 16:25:10 #310 №284635 
14608995104290.png
Что такое фотон?
Аноним 17/04/16 Вск 16:28:32 #311 №284637 
б)

Аноним 17/04/16 Вск 17:29:58 #312 №284673 
>>284637
Почему?
Аноним 17/04/16 Вск 18:15:49 #313 №284688 
>>284673
Тому що энергия hv вычисляется для одной длины волны
Аноним 17/04/16 Вск 18:37:17 #314 №284692 
Смотрю дискавери сериал Как устроена Вселенная (там 4 сезона), скоро буду знать о космосе всё.
Аноним 17/04/16 Вск 18:40:53 #315 №284694 
14609076531670.jpg
>>284692
Аноним 17/04/16 Вск 20:13:40 #316 №284712 
>>284688
Логично.
Аноним 18/04/16 Пнд 01:00:27 #317 №284794 
>>284635
Это реальный тест? Если так, то ставлю на вариант б.
Аноним 18/04/16 Пнд 01:01:36 #318 №284796 
>>284794
А, уже ответили. Ну, хоть не ошибся.
Аноним 18/04/16 Пнд 01:06:43 #319 №284799 
Что за яркая звезда сопровождает сегодня луну? Видна с 7 часов вечера ДС
Аноним 18/04/16 Пнд 02:35:27 #320 №284830 
14609361272620.jpg
>>284799
Яндекс - Планетарий ОНЛАЕН - первая ссылка.
Аноним 18/04/16 Пнд 03:23:19 #321 №284834 
>>284799
Юпитер
Аноним 18/04/16 Пнд 09:32:36 #322 №284865 
>>284830
>Яндекс
>Когда есть божественный Stellarium
Извращенец.
Аноним 18/04/16 Пнд 11:57:56 #323 №284880 
14609698767010.jpg
Сколько дней надо пробыть в невесомости после чего бы по возвращению на землю я не мог в состоянии нормально ходить?
Аноним 18/04/16 Пнд 12:00:48 #324 №284881 
>>284880
Зависит от того, выиграл ли ты в генетическую лотерею и как тренировался на земле и орбите. Но вообще вон годовой экипаж даже сам из Союза не вылез.
Аноним 18/04/16 Пнд 12:01:07 #325 №284882 
14609700674190.jpg
>>282048
>Астеройд - справочник по грамматике для специалистов аэрокосмической отрасли, астрономов и астрофизиков.
Аноним 18/04/16 Пнд 12:16:47 #326 №284887 
>>284880
Около двух недель. Но вообще очень по разному, и смотря что имеешь в виду под "нормально ходить".
Аноним 18/04/16 Пнд 14:59:39 #327 №284933 
>>280996 (OP)
Объясните по хардкору, почему омериконцы отказались от лунной программы?
Аноним 18/04/16 Пнд 15:04:30 #328 №284934 
>>284933
Дорого.
Аноним 18/04/16 Пнд 15:04:47 #329 №284935 
>>284881
>годовой экипаж даже сам из Союза не вылез
Я так понимаю, что Союз не совсем запланированно приземлился - на бок.
Аноним 18/04/16 Пнд 15:05:20 #330 №284936 
>>284934
>Объясните по хардкору
>Дорого.
Спасибо
Аноним 18/04/16 Пнд 15:19:44 #331 №284943 
>>284881
келли сказал что если бы пришлось, то вылезли бы
плюс говорит, что думал там человеки в капсулу залезут и их вытащат, а на самом деле все не так пиздато, надо отцеплять себе в этом скафандре от всего, что говорит не такая и простая задача, открывать закрывать люк, один вылез люк закрыл пересел открыл, вылез чтобы тебя уже там ухватили и вытащили.
Потом перелеты, потом медобследование, потом опять проверки туда сюда и через три дня он вполне огурцом час отвечал на вопросы. Я хз, я бы заебался в таком графике и без года на станции. Моя субъективная оценка, по крайней мере внешне, на данный момент технология уже неплохо откатана.
Да есть определенные проблемы, но уже неплохо.
Аноним 18/04/16 Пнд 15:19:47 #332 №284944 
>>284936
И бессмысленно.
Аноним 18/04/16 Пнд 15:24:41 #333 №284945 
>>284944
Почему тогда китайцы и русские туда метят? Необучаемость?
Аноним 18/04/16 Пнд 15:26:07 #334 №284946 
>>284943
А ведь на Марсе не будет спасателей и медиков
Аноним 18/04/16 Пнд 16:19:06 #335 №284973 
>>284946
об этом его тоже спросили, в этом же моменте что я уже описал.
Сказал, посидели бы подольше, оклемались чутка, плюс 1/3, вылезли бы и пошли строить коммунизм - вообщем оценил как реально.
Аноним 18/04/16 Пнд 17:17:49 #336 №284987 
>>284945
Она. К тому же у нас есть интересная программа исследований полярных областей Луны - туда американцы не летали. Но в основном конечно цель это престиж, те же отечественные луноходы принесли науки не меньше чем американская пилотируемая программа при гораздо меньших затратах.
sageАноним 18/04/16 Пнд 18:57:26 #337 №285009 
>>284987
Ну так если мы так хорошо изучили Луну, по идее должны были тоже понять, что нехуй там делать.
Аноним 18/04/16 Пнд 19:45:58 #338 №285020 
>>284987
>луноходы принесли науки не меньше чем американская пилотируемая программа
2016 год на дворе, а мифы из советских газет живут и поныне. Можно хотя бы погуглить эксперименты и инструменты Apollo, а потом Лунохода, чтобы такую ересь не нести. Очень печально, что у некоторых голова не работает, зато есть телевизор!
Аноним 18/04/16 Пнд 20:09:15 #339 №285025 
>>285020
А ещё писали, что клятые пиндосы - бесчеловечные ублюдки, раз отправили аж на Луну человека, шоб у него волосы повыпадали и детей не было, не то что правильные советы, отправляющие роботов.
Аноним 18/04/16 Пнд 20:28:20 #340 №285040 
>>284987
Давно там инфа о сейсмозондировании луноходами появилась?
Аноним 18/04/16 Пнд 20:32:02 #341 №285041 
>>284259
Ну вот китайцы на эмбрионы мшек вырастили
https://lenta.ru/news/2016/04/18/embryo/
Аноним 18/04/16 Пнд 20:32:18 #342 №285042 
>>285041
>мшек
мышек
фикус
Аноним 18/04/16 Пнд 20:53:50 #343 №285048 
Где-то видел проекты таких себе "лунных ступ" вместо роверов для Аполлонов. Может есть у кого больше инфы?
Аноним 18/04/16 Пнд 21:57:57 #344 №285069 
>>284259
Птенцов выводили.
https://www.youtube.com/watch?v=swBgv4_Ka2s
Аноним 19/04/16 Втр 01:13:25 #345 №285132 
>>284945
Мурриканцы уже там побывали и кому надо все доказали. А русские и китайцы еще нет. Вот и метят.
А хотя что значит "метят". Американцы тоже метят. Но это скорее так, мечты о том, что когда-нибудь мы там будем, лет через 15. Нет, через 20. Через 20 точно. На через 15 денег сейчас нет, а через 20 - точно будем. Максимум - через 25. Прокрастинация такая государственного масштаба.
Аноним 19/04/16 Втр 01:14:06 #346 №285133 
>>285020
Толи дело швитые марс-роверы!
sageАноним 19/04/16 Втр 01:37:48 #347 №285139 
Вы заблудились? Вам показать дорогу до пораши?
Аноним 19/04/16 Втр 04:17:21 #348 №285151 
Как же заебала протечка из пораши с лунным заговором.
Аноним 19/04/16 Втр 12:34:42 #349 №285188 
>>285185
Русские в космосе всегда были и остаются первыми.
Циолковский мечтал о космосе и изучил принцип реактивного движения. Потом Королев реализовал некоторые его мечты, первым запустив в космос спутник, потом животных, человека, женщину, первым изобрел ракету и корабль для лунной программы, которые используются по всему миру до сих пор.
В плане антигравитации русские тоже были первыми.

А для америки космос всегда был как шоу. Что у них было реальное? Шатлы только.
Аноним 19/04/16 Втр 12:40:10 #350 №285190 
>>285188
>человека, женщину
лал
>антигравитации
что бля?
Аноним 19/04/16 Втр 12:40:48 #351 №285191 
>>285151
Хорошо. Если правильно ответишь на простой вопрос, то я уйду.

Если у Америки была успешная программа, то где оставшиеся от нее технологии? Почему сейчас в космос летают на протонах, а не на сатурнах? Почему космонавтов возят на МКС в союзах, а не в аполлонах?
Аноним 19/04/16 Втр 13:05:58 #352 №285195 
14610603588840.jpg
Как узнать когда спутник пролетит над моим мухосранском, чтобы сфотать его из космоса, для гугл и яндекс карт? Хочу нарисовать хуй на песке
Аноним 19/04/16 Втр 13:08:28 #353 №285197 
>>285195
http://heavens-above.com/
Аноним 19/04/16 Втр 13:09:01 #354 №285198 
>>285191
Ты, когда такси в булочную заказываешь, просишь подать тебе лимузин?
мимодругойанон
Аноним 19/04/16 Втр 13:10:50 #355 №285199 
>>285188
>летают на протонах
И кто у тебя летает на Протонах, говно залетное? Съебись уже нахуй.
Аноним 19/04/16 Втр 13:16:58 #356 №285201 
>>285199
Можно подумать что не летают. Протоны в строю до сих пор, а вот сатурны не используют. Почему? Сатурн такой плохой что даже не может нормально вывезти груз на орбиту?
Аноним 19/04/16 Втр 13:18:00 #357 №285202 
>>285195
http://www.sat.belastro.net/heavensat.ru/
Аноним 19/04/16 Втр 13:25:43 #358 №285205 
>Почему?
Потому что есть другие ракеты. Нахуй нужен сверхтяж для Луны в обычных миссиях? Нахуя? Ты за картохой в магазин на Камазе ездишь, полуебок?
Аноним 19/04/16 Втр 13:31:20 #359 №285207 
>>285205
Ну хоть один модуль МКС им бы вывели или еба-телескоп на орбиту или крупный зонд в сторону Марса. Зачем прятать успешную ракету и не использовать?
Аноним 19/04/16 Втр 13:35:30 #360 №285209 
>>285207
Вывели скайлаб - недоМКС, здоровенную одномодульную йобу. К тому времени как МКС запускали Сатурн уже пару десятков лет как с производства сошел. Ты бы еще спросил почему сейчас вместо Энергии - готового сверхтяжа невнятную тяжелую модификацию Ангары пилят в год по чайной ложке.
Впрочем, ты же лунный заговорщик, ты еще как-нибудь вывернешься.
Аноним 19/04/16 Втр 13:40:20 #361 №285211 
>>285207
>Ну хоть один модуль МКС им бы вывели или еба-телескоп на орбиту или крупный зонд в сторону Марса.
Нахуя? Полоумный? На кой хуй пилить это порождение лунной гонки для вывода на орбиту грузов, которые с успехом выводятся другими носителями?
Аноним 19/04/16 Втр 15:59:57 #362 №285236 
>>285198
он требует лимузин, шампанского, шлюх, клоунов, медведей и чемодан баксов.
совсем другой
Аноним 19/04/16 Втр 16:06:11 #363 №285238 
Температура урануса?
Аноним 19/04/16 Втр 16:07:35 #364 №285239 
>>285207
Во же неугомонный уебан. Это все равно что делать операцию на сердечном клапане с помощью мачете, считать 2+2 на квантовом суперкомпьютере, ездить за хлебом на Феррари, жарить шаурму в доменной печи, ну ты понял надеюсь.
Аноним 19/04/16 Втр 16:09:57 #365 №285240 
>>285238
-224 минимальная
Аноним 19/04/16 Втр 17:44:33 #366 №285256 
>>285188
>Русские в космосе всегда были и остаются первыми.
>луноходы принесли науки не меньше чем американская пилотируемая программа
Охуительные истории ИТТ.
Аноним 19/04/16 Втр 18:07:22 #367 №285262 
>>285238
В недрах, или верхних слоёв атмосферы?
Аноним 19/04/16 Втр 21:56:34 #368 №285332 
>>283931
Почему нз с первой гифки создает впечатления наблюдения за маяком. Разве вся поверхность неравномерна освещена?
Аноним 19/04/16 Втр 22:05:16 #369 №285335 
>>285332
Может это джет?
sageАноним 19/04/16 Втр 22:06:01 #370 №285336 
14610927613030.gif
>>285332
Потому что это и есть маяк, только в радиодиапазоне.
Аноним 19/04/16 Втр 22:49:46 #371 №285379 
>>285336
Все равно не понимат. Я вот представляю себе раскаленный и равномерно освещенный шар, вращающийся с огромной скоростью, но почему образуется фокус непонятно.
Аноним 19/04/16 Втр 22:58:06 #372 №285385 
14610958861410.jpg
>>285379
Потому что направленное излучение идет из магнитных полюсов.
Аноним 19/04/16 Втр 22:58:17 #373 №285386 
>>285379
https://ru.wikipedia.org/wiki/Релятивистская_струя
Аноним 20/04/16 Срд 09:00:57 #374 №285543 
>>285386

Интересная, кстати, тема, и какое-то простое явление (не эфир, блять!) остается еще не открытым:
https://ru.wikipedia.org/wiki/Релятивистская_струя#.D0.94.D0.B0.D0.BB.D1.8C.D0.BD.D0.B5.D0.B9.D1.88.D0.B5.D0.B5_.D0.B8.D0.B7.D1.83.D1.87.D0.B5.D0.BD.D0.B8.D0.B5_.D1.80.D0.B5.D0.BB.D1.8F.D1.82.D0.B8.D0.B2.D0.B8.D1.81.D1.82.D1.81.D0.BA.D0.B8.D1.85_.D1.81.D1.82.D1.80.D1.83.D0.B9
>При самых первых попытках объяснения сверхсветового движения с помощью релятивистского направленного потока частиц возникло осложнение: удивительно большая доля компактных источников показывала сверхсветовое движение, в то время как на основании простых геометрических доводов получалось, что только несколько процентов таких объектов должно быть случайно ориентировано почти вдоль линии зрения.
Аноним 20/04/16 Срд 10:11:38 #375 №285554 
>>285201
Тебе пиццу небось на КАМАЗе привозят? Или ты тралишь?
Аноним 20/04/16 Срд 13:26:15 #376 №285599 
14611479756450.png
>>280996 (OP)
Вот как. Гармония
Аноним 20/04/16 Срд 16:54:04 #377 №285646 
Нужна софтинка, в которой легко можно создавать хуитки на произвольной орбите например Земли, и наблюдать-катать параметры орбиты, период там, скорость и всё такое. KSP с HE был бы норм, но там система левая и вообще это игра.
Аноним 20/04/16 Срд 16:54:35 #378 №285647 
>>285646
Можно что-то простое, без йоба-моделирования влияния пролетающего шарика космического говна.
Аноним 20/04/16 Срд 17:40:34 #379 №285667 
>>285646
https://gitard.github.io/kosmach/Космач.html
Аноним 20/04/16 Срд 17:44:09 #380 №285669 DELETED
>>285543
>не эфир, блять!
У тебя и вакуум, имеющий свойства, не эфир, реляв.
Аноним 21/04/16 Чтв 02:08:50 #381 №285829 
Правда что в новый телескоп можно будет увидеть обитателей близких планет?
Аноним 21/04/16 Чтв 02:22:20 #382 №285832 
>>285829
>новый телескоп
Да и в б/у телескоп можно увидеть, главное правильные вещества употребить. А почему ты уверен, что на близких планетах кто-то обитает?
Аноним 21/04/16 Чтв 02:29:38 #383 №285833 
>>285832
Ну когда новый телескоп прилетит будет понятно. Но я думаю не Зоя столько миллиардов планет т планетоидов. Где-то должны жить существа.
Аноним 21/04/16 Чтв 09:26:47 #384 №285860 
>>285833
Явно не Зоя.
Аноним 21/04/16 Чтв 10:24:35 #385 №285870 
Может где-то подо льдами Европы наши братья по разуму ждут освобождения из ледяного плена
Аноним 21/04/16 Чтв 10:46:33 #386 №285877 
14612247938000.jpg
>>285870
>братья по разуму
Аноним 21/04/16 Чтв 11:06:46 #387 №285894 
>>280996 (OP)
Школьник кун вкатывается в тред с тупым вопросомвсе как обычно:

Как могут существовать объекты, диаметр которых превышает ~300 000 км(скорость света)? То же наше солнце, судя по википедии имеет диаметр 1 392 684 км, как оно остается при этом цельным? Мне трудно как-то правильнее сформулировать свой вопрос правильнее, но я надеюсь вы поняли, что я имею ввиду.
Аноним 21/04/16 Чтв 11:11:01 #388 №285897 
>>285894
Для начала скажи мне, зачем ты сравниваешь диаметр и скорость?
В чём ты видишь проблему существования таких объектов?
Аноним 21/04/16 Чтв 11:13:53 #389 №285900 
>>285897
Ну вот, правильнее не смог свой вопрос сформулировать, так-с, сейчас придумаю как попроще его сказать:

Ну вот, допустим, есть палка длинной в две световые секунды, она же по идее должна схлопнутся под собственной массой, так? Или вот допустим, потратил много энергии чтобы наклонить один конец этой палки, получается что только через две секунды другой конец палки наклонится, так ведь?
Аноним 21/04/16 Чтв 11:20:31 #390 №285905 
>>285900
>должна схлопнутся под собственной массой
Нет.
>только через две секунды другой конец палки наклонится
Нет.
Аноним 21/04/16 Чтв 11:21:57 #391 №285907 
>>285905
А как это работает? Поясни пожалуйста. Я просто не понимаю, в чем тут фокус и как это работать должно.
Аноним 21/04/16 Чтв 11:25:16 #392 №285909 
>>285900
Нет, схлопываться под своей массой она не должна - с чего бы? Масса, скорее всего, невелика, плотность тоже.
Скорость распространения толчка от одного конца палки до другого - скорость звука в материале из которого сделана палка. И эти колебания имею свойство затухать. Так что далеко не факт, что второй конец палки вообще пошевелится, как бы ты не теребонькал свой конец и как бы двусмысленно это не звучало
Аноним 21/04/16 Чтв 11:29:14 #393 №285913 
>>285909
Так вот в чем прикол, если распространяется со скоростью звука, то почему одна часть Солнца под действием чего-то не отваливается и не отлетает от общей массы, то есть: Солнце же постоянно в движении, оно вокруг центра галактики постоянно движется, как оно остается цельным при всем этом движении?
Аноним 21/04/16 Чтв 11:31:47 #394 №285915 
>>285913
Окей. Хуй с пальцем палку со звездой не сравнивай, да?
Гравитация с какой скоростью распространяется?
Аноним 21/04/16 Чтв 11:33:48 #395 №285916 
>>285915
Ладно, короче ясно, больше книжек надо читать, ну спасибо, что потратил на меня свое время

>Гравитация с какой скоростью распространяется?
Вроде как со скоростью света, но мне рассказывали, что это не доказано и вполне может превышать эту скорость.
Аноним 21/04/16 Чтв 11:37:22 #396 №285917 
>>285646
Universe Sandbox (2)?
Аноним 21/04/16 Чтв 11:41:55 #397 №285918 
>>285916
Всегда пожалуйста.

>Вроде как со скоростью света, но мне рассказывали, что это не доказано и вполне может превышать эту скорость.
Напротив - обратное не доказано. Пока все имеющиеся наблюдения, измерения и их интерпретации говорят о том, что скорость гравитации близка или равна c
Аноним 21/04/16 Чтв 12:05:05 #398 №285922 
14612295053830.jpg
>>285907
>схлопнутся под собственной массой
Это значит гравитация преодолевает все остальные силы. В Солнце происходит термоядерный синтез, в результате которого выделяется энергия, создающая внутреннее давление. До тех пор пока гравитация слабее него, Солнце не схлопывается.
Это правило в принципе действует на все остальное. В любом стабильном объекте есть внутренняя сила, противостоящая гравитации. Световые секунды здесь не причем.
Аноним 21/04/16 Чтв 17:03:48 #399 №286003 
Если люди ступят на Марс, то мы неизбежно заразим планету микроорганизмами? Мы же не сможем продезинфицировать всех людей, а там где люди - там и различная живая микроорганика?

Как можно будет после высадки на Марс искать там следы жизни, если мы сами её туда занесем?
Аноним 21/04/16 Чтв 17:04:50 #400 №286004 
>>285870
А может подо льдами Европы плещется раствор серной кислоты)))
Аноним 21/04/16 Чтв 17:18:04 #401 №286009 
>>286003
Только эта органика должна что-то жрать и обитать в похожих условиях. Поэтому вряд ли она сможет выжить на Марсе вне мест обитания людей. Хотя отдельные организмы, возможно и смогут приспособиться, если найдут замену привычной хавке.
Аноним 21/04/16 Чтв 17:27:21 #402 №286011 
>>286009
А ничего, что, например, на поверхности МКС были найдены живые микроорганизмы морского происхождения?
Что помешает этим вездесущим малявкам добраться до Марса и выжить там? Ведь достаточно одного-двух, что смогут выжить и размножиться. И всё - привет, жизнь на Марсе.
Аноним 21/04/16 Чтв 17:29:13 #403 №286012 
>>286003
Если бы там были условия для существования жизни, то она бы там уже зародилась. Все микроорганизмы, что туда будут занесены умрут в неблагоприятной среде. В конце концов следы жизни имеют возраст, поэтому отличить говно от палки будет не трудно.
Аноним 21/04/16 Чтв 17:31:11 #404 №286014 
>>286011
Туда уже отправили уйму аппаратов, но вот что-то марс до сих пор красная планета, а не зеленая.
> на поверхности МКС были найдены живые микроорганизмы
А еще на МКС находили от трех до шести особей Homo Sapiens, и что?
Аноним 21/04/16 Чтв 17:35:31 #405 №286015 
>>286004
Ну думаю даже если так какая-то живность вроде червей или миниакул там все же присутствует
Аноним 21/04/16 Чтв 17:36:51 #406 №286016 
>>286014
>
Туда уже отправили уйму аппаратов, но вот что-то Марс до сих пор красная планета, а не зеленая.
Аппараты перед отправкой собирают в "чистой комнате" и вообще содержат в условиях, близким к стерильным, что существенно уменьшает вероятность выживания там каких-либо микроорганизмов. В противоположность аппаратам, человек носит микроорганизмы в себе (между прочим, около двух килограмм). Какова вероятность того, что этими организмами может быть загрязнена поверхность чужой планеты с известными последствиями? Явно не нулевая.
Кроме того, процесс озеленения, по-видимому, представляется вам каким-то магическим процессом - пуф! - и Марс зелёный. Откуда вы знаете что уже сейчас там нет жизни, занесённой земными аппаратами?
Аноним 21/04/16 Чтв 17:37:07 #407 №286017 
>>286016
Блядь, разметку проебал
Аноним 21/04/16 Чтв 17:37:07 #408 №286018 
>>286011
>А ничего, что, например, на поверхности МКС были найдены живые микроорганизмы морского происхождения?
Ты не путай тёплое с мягким. "Нашли микроорганизмы" и "МКС покрылась микроорганизмами и зацвела" не одно и тоже. Они там могут на обшивке сидеть в полудохлом состоянии хоть десять лет, жрать там все равно нечего и для размножения никаких условий нет. Более того совершенно не ясно как долго они там тусуют, может они там ежедневно обновляются, а предыдущие дохнут.
Аноним 21/04/16 Чтв 17:40:41 #409 №286019 
>>286018
>Они там могут на обшивке сидеть в полудохлом состоянии хоть десять лет, жрать там все равно нечего и для размножения никаких условий нет. Более того совершенно не ясно как долго они там тусуют, может они там ежедневно обновляются, а предыдущие дохнут.
С этими аргументами спорить трудно, согласен. Но, тем не менее, вероятность занесения жизни на Марс, как мне думается, присутствет
Аноним 21/04/16 Чтв 17:42:20 #410 №286020 
>>286016
>Аппараты перед отправкой собирают в "чистой комнате" и вообще содержат в условиях, близким к стерильным
А собирают их люди, внутри которых 2кг микроорганизмов.
>Откуда вы знаете что уже сейчас там нет жизни, занесённой земными аппаратами?
Я не знаю. Но я просто не понимаю твоего изначального вопроса.
>Как можно будет после высадки на Марс искать там следы жизни, если мы сами её туда занесем?
Ты имеешь ввиду технологический аспект вопроса, моральный или это вообще риторический вопрос?
Аноним 21/04/16 Чтв 18:02:08 #411 №286023 
>>286012
Ты не прав.
Аноним 21/04/16 Чтв 18:04:36 #412 №286024 
>>286019
безусловно, но без подходящих условий распространения она будут прокрастинировать на остатках своих предшественников, эдакими колониями оазисами.
Я тут както говорил о бесполезности этих процедур дезенфикации.
Но я чето передумал после вашего диалога, представив что там во льдах можно найти следы-остатки-микроорганизмы во льдах, которые были занесены туда десятки миллионов лет назад и не окаменели. Хм, я чето обратно передумал, но вообще хотелось бы конечно их зацепить и не проебать такую возможность, если она все таки есть. Шанс не велик однако, да еще текущими инструментами, но и действительно одно от другого можно будет отделить, если что. Так что пох, главное чтобы наши там не съели все.
Аноним 21/04/16 Чтв 18:05:19 #413 №286026 
Кароче, спейсачеры, вопрос возник. Вот имеется у нас черная дыра. Вокруг нее летает аккреционный диск, и внутренние слои постепенно падают в ЧД. Вот мне интересно, для стороннего наблюдателя этот разогретый газ, падающий в черную дыру будет постепенно замедляться, и никогда не достигнет горизонта событий, в то время, когда на самом деле этот газ уже давно преодолел его, ведь так? Тогда следующий вопрос: это газ поидее излучает дохуя света, ведь он очень горячий, но как его излучение будет изменяться в зависимости от его близости к горизонту событий для того же стороннего наблюдателя? Будет ли он излучать вечно?
Аноним 21/04/16 Чтв 18:05:56 #414 №286027 
>>286016
я вообще задал вопрос о пилотируемой экспедиции, которую нельзя 100% стерилизовать.
На Марсе ебучие песчаные дьяволы и бури могут разнести микроорганизмы на широкую площадь. То есть в каких-то местах они впадут в гибернацию, но не факт, что они не найдут какую-то комфортную зону под камнем или в трещине, где размножатся и их опять унесет ветром куда-то и так далее.
Аноним 21/04/16 Чтв 18:06:04 #415 №286028 
>>286026
>газ уже давно преодолел его, ведь так?
нет.
Аноним 21/04/16 Чтв 18:06:52 #416 №286029 
>>286023
А если разбавить словами "вероятно, некоторые, наверное"?
Аноним 21/04/16 Чтв 18:08:07 #417 №286030 
>>286028
Чому? Что ему помешает?
Аноним 21/04/16 Чтв 18:10:14 #418 №286031 
>>286027
>где размножатся и их опять унесет ветром куда-то
> найдут какую-то комфортную зону под камнем или в трещине, где размножатся
это и есть пути распространения, если они отсутствуют, не означает что заражения не будет, но распространяться оно практически не будет.
В принципе вся эта мелочь, не так чтобы дохуя прочная, она числом берет. Те шанс сойти на ноль вполне себе не нулевой, если условия не очень.
Аноним 21/04/16 Чтв 18:10:23 #419 №286032 
>>286027
В чем вопрос то?
Как стерилизовать? Или что?
Аноним 21/04/16 Чтв 18:13:17 #420 №286034 
>>286030
ничто, для простоты, но с точки зрения внешнего наблюдателя это ничего не меняет. Те можешь сказать, что этому мешает внешний континуум, в котором этот наблюдатель находится.
Аноним 21/04/16 Чтв 18:16:05 #421 №286036 
>>286034
Ну меня не это интересовало, в основном я спрашивал про то, что со стороны наблюдателя весь газ будет оседать на горизонте событий. А так же как будет выглядеть излучение этого осевшего на горизонт событий газа.
Аноним 21/04/16 Чтв 18:29:32 #422 №286038 
>>286032
Вопрос нахера лететь, если можно еще 40 лет изучать с орбиты. На том же Фобосе можно сесть и роботами управлять на поверхности Марса. Роботов хотя бы можно стерилизовать нормально, да и по способностям они будут как человек уже к 2040 году.
Аноним 21/04/16 Чтв 18:45:16 #423 №286039 
>>286038
> 40 лет
Тогда уже денег никто на это давать не будет.
Аноним 21/04/16 Чтв 18:51:46 #424 №286041 
>>286026
>>286036

>как его излучение будет изменяться
Будет сползать в сторону удлинения волны, по мере приближения к горизонту событий, так что никакого падения непосредственно ты не увидишь. То есть видимый диапазон сначала переползет в красный спектр, потом в инфракрасный, в микроволны, потом в сверхдлинный радио-диапазон.


>>286019
Я, заметь, полностью не отрицал, просто мне это представляется маловероятным.
Аноним 21/04/16 Чтв 18:53:20 #425 №286042 
>>281642
Цвет реголита – серо-коричневый и довольно тёмный, но когда Солнце интенсивно светит прямо на реголит виден яркий рассеянный свет, который воспринимается как белый. Если же уменьшить чувствительность или привезти реголит на Землю, будет видно, что он имеет в целом серо-коричневый цвет.
Аноним 21/04/16 Чтв 18:57:28 #426 №286043 
>>286026
>для стороннего наблюдателя этот разогретый газ, падающий в черную дыру будет постепенно замедляться, и никогда не достигнет горизонта событий
>как его излучение будет изменяться в зависимости от его близости к горизонту событий для того же стороннего наблюдателя Будет ли он излучать вечно?
Вечно ничто не излучает. Для стороннего наблюдателя газ постепенно краснеет, тускнеет и исчезает из виду.
Объект пересек горизонт, а что там будет со светом ему все равно. Сторонний наблюдатель будет видеть, как длина волны излученного газом света увеличивается, пока не станет такой большой, что глаза стороннего наблюдателя перестанут этот свет улавливать.
Аноним 21/04/16 Чтв 18:59:20 #427 №286044 
>>282404
У РД-170 на минуточку мощность как у Саяно-Шушенской ГЭС А у РД-180 как у СШГЭС после неудачной отправки одного гидроагрегата в космос
Аноним 21/04/16 Чтв 19:12:45 #428 №286046 
>>286044
сложи их все вместе чтобы они выработали столько же энергии, после этого приходи, будем сравнивать.
Аноним 21/04/16 Чтв 19:20:00 #429 №286048 
>>286044
Какая мощность? Мощность сгорающих газов или полезная мощность?
Аноним 21/04/16 Чтв 19:31:59 #430 №286050 
>>286004
Как выглядит "ледяная корка" серной кислоты? Механизмы ее кислоты под водяным льдом? Почему они не смешиваются? Почему такая агрессивная среда, за миллиарды лет, еще не прореагировала/испарилась в открытый космос?

Набор простейших вопросов.
Аноним 21/04/16 Чтв 19:36:03 #431 №286051 
>>286004
Лёд растворяется в серке на раз.
Аноним 21/04/16 Чтв 19:36:50 #432 №286052 
>>286048
это там практически одно и тоже
Аноним 21/04/16 Чтв 20:04:12 #433 №286063 
>>286050
>>286051
Мэтью Пасек с сотрудниками из Университета Южной Флориды на основании анализа данных о составе поверхностного слоя Европы и скорости диффузии кислорода в подлёдный океан сделали вывод, что в нём слишком велика концентрация серной кислоты и океан непригоден для жизни.

Серная кислота в океане Европы образуется в результате окисления кислородом минералов ядра спутника, содержащих серу. Прежде всего — сульфидов металлов. Согласно расчётам авторов статьи, показатель кислотности рН воды подлёдного океана составляет 2,6 единицы — это на порядок больше, чем в сухом красном вине. Углеродная жизнь в таких средах, по мнению астробиологов, крайне маловероятна или вовсе невозможна.


Подробнее см.: http://www.nkj.ru/archive/articles/20780/ (Наука и жизнь, На Европе жить довольно кисло)
Аноним 21/04/16 Чтв 20:38:59 #434 №286092 
>>286052
Полезная мощность двигателя на стенде равна нулю. А мощность двигателя на орбите юпитера в несколько за превосходит мощность химического взаимодействия. Вот я и спрашиваю, какая мощность равна СШГЭС
Аноним 21/04/16 Чтв 20:41:34 #435 №286096 
>>286063
А есть какие-то теории насчет других форм жизни? Я где-то читал, что фтор подобно углероду может создавать стабильные большие молекулы или типо того. Есть ли неуглеродные формы жизни на земле?
Аноним 21/04/16 Чтв 20:51:45 #436 №286103 
>>286092
>Полезная мощность двигателя на стенде равна нулю
зависит от стенда, будет дуть в турбину, к которой подключен генератор, который подключен к лампочкам и гидропоникесами знаете с чем и полезная мощность превысит на 150 процентов саяно шушерской.
>А мощность двигателя на орбите юпитера в несколько за превосходит мощность химического взаимодействия.
нет, потенциальная энергия тут тоже не причем.
Аноним 21/04/16 Чтв 21:17:50 #437 №286110 
>>286096
>фтор
Слишком активен, не слишком распространен.
>Есть ли неуглеродные формы жизни
Нет, и маловероятны.
Аноним 21/04/16 Чтв 21:34:09 #438 №286121 
>>282657
Да может, почему нет.

Только жизнь к радиации будет поболее устойчивой. Систем репарации будет больше.

И приливный захват не проблема если будет вращаться вокруг своего гиганта быстро.
Аноним 21/04/16 Чтв 22:13:00 #439 №286146 
>>286103
>нет, потенциальная энергия тут тоже не причем.
Да, не причем. Но факт остается фактом, прирост кинетической энергии ракеты на единицу времени (а именно это для ракеты является полезной мощностью, как я полагаю) на большой скорости может многократно превосходить мощность сгорания газов. В семидесятых годах прошлого века в СССР одного инженера чуть в психушку не посадили, за то что он придумал, как нагревать воду в трубах центрального с КПД больше 1, потом началась перестройка и он эмигрировал в США, сейчас эта технология запатентована, и называется динамическое отопление. Со стороны выглядит, как нарушение законов термодинамики.
Аноним 21/04/16 Чтв 22:16:38 #440 №286148 
>>286110
Погуглил слегка: в условиях большой температуры и при наличии серной кислоты может существовать кремнеевая жизнь. Для нее нужны большие энергии, а в качестве растворителя подходит серная кислота. Такие дела.
Аноним 21/04/16 Чтв 22:17:01 #441 №286149 
>>286146
> потенциальная энергия
эк я себе пути для маневра отрезал, ну ладно хуй с ним
Аноним 21/04/16 Чтв 22:41:47 #442 №286166 
>>286148
>Для нее нужны большие энергии, а в качестве растворителя подходит серная кислота
Чужой, случайно, не является кремниевой формой жизни?
Аноним 21/04/16 Чтв 23:03:24 #443 №286176 
>>286148
Элементы третьего периода — кремний и фосфор — формируют стабильные цепи только при чередовании с кислородом, причём гораздо более массивные и склонные к образованию однообразных сетчатых (двухмерных) полимеров. Кроме того, эти элементы в цепи не обладают способностью образовывать кратные пи-связи, совершенно необходимые для сборки плоских структур, поэтому «чисто-силоксановая» жизнь невозможна — совсем без углерода обойтись не удастся и максимум возможного здесь — примесь кремния в гетероорганических молекулах: органосилоксанах и, возможно, органосилатранах.
Аноним 22/04/16 Птн 00:05:11 #444 №286197 
Можно ли получить плазму в домашних условиях?
Аноним 22/04/16 Птн 00:56:29 #445 №286223 
>>286197
Купи неоновую лампочку и любуйся, это плазма. Продаются большие, их можно в 220 вкручивать, там различные композиции в них даже.
Да можно, искра между контактами это тоже плазма - всякие тесла фанаты за большим.
Вакуумный насос и литровая банка с контактами, и в розетку гонять плазму.
И тд, можно короче, это не такая большая проблема.
Аноним 22/04/16 Птн 03:21:57 #446 №286249 
14612845170610.jpg
>>286197
Свечку зажги.
Аноним 22/04/16 Птн 03:25:21 #447 №286250 
Я нюфаг, обьясните просто и ясно, почему идея с микрозондами не взлетит.
нет, в тоед не пойду, там содомия
Аноним 22/04/16 Птн 08:58:19 #448 №286270 
Поясните за САС "Бурана". Я правильно понимаю, что, как и у шатлов, спасение на начальном этапе полёта было невозможно. Вроде как предполагали ставить катапульты, но потом от них отказались. Просто не пойму, как в случае неразвала СССР и последующего начала пилотируемых полётов это бы осуществлялось, ведь уже был печальный опыт "Челленджера".
Аноним 22/04/16 Птн 09:04:44 #449 №286271 
>>286249
это ненастоящая плазма, дятел
Аноним 22/04/16 Птн 09:10:36 #450 №286273 
>>286271
Ты понимашь, что понятия настоящая и ненастоящая плазма есть только в твоём воображении?
Аноним 22/04/16 Птн 09:18:53 #451 №286274 
>>286273
Температуру ионизации газов гугли, свиноеб
Аноним 22/04/16 Птн 09:23:38 #452 №286277 
>>286274
Может ты определение плазмы загуглишь для начала? Тебе 15 лет что ли, дурачек?
Аноним 22/04/16 Птн 09:56:36 #453 №286281 
>>286250
1) Сложности миниатюризации - но тут проблема чисто инженерная, может и получится.
2) Сложности передачи информации - вояджеры наполовину состоят из трехметровой тарелки и все равно передают инфу в час по чайной ложке - сигнал-шум никто не отменял, миниатюризация тут не поможет.
3) И последнее, но не по значению - протрутся впизду эти зонды межзвездной средой, если скорость большая будет. А если будет небольшая, то нахуя огород городить, современные технологии и так позволяют отправить зонд, который долетит до ближайших звезд всего за несколько тысяч лет.

И собственно вопрос "нахуя?" - Нахуя этот огород городить, если массив инфракрасных телескопов в одной из точек Лагранжа за смешные деньги (несколько десятков миллиардов долларов) позволит получить любые потребные данные об экзопланетах?
Аноним 22/04/16 Птн 10:02:23 #454 №286282 
>>286063
>Впрочем, биологам известны примеры существования микроорганизмов на Земле и в гораздо более суровых условиях (о достаточно крупных существах речь вообще не идёт). В связи с этим Мэтью Пасек цитирует данные анализа воды реки Рио-Тинто в Испании, которая имеет глубокий красный цвет и рН от 1 до 5 из-за стока в реку дренажных вод, образующихся при добыче железной руды. И там живут микроорганизмы, использующие для своего метаболизма железо и серу.
Аноним 22/04/16 Птн 10:05:34 #455 №286283 
>>286282
И да, к слову, видел данные что весьма возможно что подледный океан на Европе вообще не имеет связи с каменным ядром из за толстого слоя прессованного льда. Что, впрочем, мешает возможной жизни куда больше. Ну ничего, остаются Энцелад (где океан точно имеет связь с ядром), Ганимед и прочая параша.
Да, я подледноокеаносектант.
Аноним 22/04/16 Птн 10:30:23 #456 №286287 
>>286283
А насичет Плутона что думает ваша секта?
Аноним 22/04/16 Птн 10:53:54 #457 №286293 
14613116344120.png
Анон, посмотри на какой пиздец я наткнулся
http://vk.com/club12072610
Они яростно доказывают, что америка вообще не летает в космос, и что Илон не мог посадить ступень, да и тоже не отправлял ракеты в космос. При этом мусорка абсолютно не понимает матчасти.
Я не верю, что такие долбоебы живут в наше время. Как их проучить можно?

Как же у меня жопа сгорела
sageАноним 22/04/16 Птн 11:00:30 #458 №286296 
>>286293
Если они не понимают матчасти, то не получится и общаться с ними.
в группе 237 человек, это мелочовка какая-то. Понимаю если бы в группе был миллион человек, тогда можно было бы волноваться и гореть, но 237 дебилов это не так уж и много. В США гораздо больше сторонников лунного заговора и идей, что США ничего не запускают, чем в России.
Аноним 22/04/16 Птн 11:03:48 #459 №286297 
>>286296
Как можно быть настолько тупым, что бы лезть в такую тему, но при этом не понимать в ракетах и полетах НИЧЕГО, и при каждом ПРАВДИВОМ слове банить нахуй. Причем если пишешь с реальными фактами. Блядь, на человек, который в KSP поиграет месяцок и то больше понимать будет, чем долбоебы эти.

Давно я так не бугуртил.
Аноним 22/04/16 Птн 11:04:22 #460 №286298 
>>286287
А хуй его знает, про Плутон я данных еще не видел. Хотя приливные силы должны приличные быть от Харона для разогрева (говорю это основываясь ни на чем).
>>286293
Тут вроде говорят Маск летал, только ДОРОХА. https://vk.com/club12072610?w=wall-12072610_9213%2Fall
С цифрами у них интересное обращение.
А вообще ребята в целом как-то странно пишут, я не совсем понимаю на каком это языке и что пытаются донести. У меня лично пукан устоял, хотя он у меня тренированный (закоптился сильнее чем двигатели на фальконе) - лунных заговорщиков я перечитал немало.
Аноним 22/04/16 Птн 11:06:58 #461 №286299 
>>286297
чувак, у них аргументация уровня плоскоземлян - те вообще говорят, что видео посадки фалькона это реверс перемотка взлета с добавленным CGI, а если ты с фактами и цифрами к ним будешь аргументировать, то тебя назовут агентом НАСА и закидают их версиями +15. Интернет позволяет людям создавать самодостаточные замкнутые манямирки, в которых они варятся, с этим ты ничего поделать не сможешь.
Аноним 22/04/16 Птн 11:11:07 #462 №286301 
>>286298
Они яростно доказывают, что посадить ступень не могли посадить, что на марсе роверы не были, на луне никого не было. Ну да блядь, Н-1 прикрыли, вот теперь решили отыгрываться на простых людях, которые любят ракеты и космос.
Я могу понять лунный заговор, лендер и картона и т.д., потому что в тот момент СССР проиграл гонку. Но блядь, в 2016 году доказывать, что посадить ступень не могли...
А еще банят при любом случае, вообще. Ладно, я ПОСТАРАЮСЬ забыть про этот пиздец.
Аноним 22/04/16 Птн 11:18:14 #463 №286305 
>>286298
>Маск-Фантомаск — суперфейк США? Илон Маск — отнюдь не «золотой мальчик», за кого он себя выдает. В действительности это хорошо разрекламированная подставная фигура, с целью распила американского бюджета, и прикрытия технологической деградации в Соединенных Штатах. Секрет его успеха — могущественные покровители, раскрутившие Маска в годы президентства Дж.Буша.

Все, беру свои слова обратно. Я даже читать дальше не стал, стул уже начал тлеть.
>>286301
Я тут недавно видел подробные опровержения полетов Джемини. Не у них, вроде где-то у нас в тредах ссылку скидывали.
sageАноним 22/04/16 Птн 11:19:12 #464 №286306 
14613131527980.png
>>286293
Считаю тупым вопросом вот это:
>как так получилось, что такие долбоебы живут в наше время?
А так. В наше время существуют долбоебы, доказывающие что земля плоская. Это вполне естественно и нормально, не все люди обучаемы в силу генетической лотереи, воспитания, и чёрт знает чего ещё.
>Как их проучить можно?
Повысив коэффициент образованных людей. Пока не-долбоёбов подавляюще больше - долбоёбы не высовываются. А уж тем более не вбрасывать их на спейсач, нахуя тут-то они. В Бодрияровском смысле (см. "Войны в заливе не было") они даже правы. Естественно, такое им даже в голову не приходит, но это не повод тащить это говно на спейсач. Сажи тебе.
Аноним 22/04/16 Птн 11:27:29 #465 №286311 
>>286306
Слыш ты, симулякр эпифеноменовый, ты неправ. Стоит вообще создать отдельный коспирологического бугурта тред.
Аноним 22/04/16 Птн 11:36:25 #466 №286314 
>>286311
Есть https://2ch.pm/zog, там более 3 космических тем, там и буггурти.
Аноним 22/04/16 Птн 11:37:04 #467 №286315 
>>286314
Запятую удали конечно же
Аноним 22/04/16 Птн 11:42:36 #468 №286317 
>>286314
В курсе. Так это зогач, а это космач. Там конспироложить, сюда приносить и бугуртить. Разделение труда.
Аноним 22/04/16 Птн 11:43:49 #469 №286318 
>>286317
Конспиролухи прибегут в тред и начнут серить и здесь вместо своего загона.
Аноним 22/04/16 Птн 11:43:52 #470 №286319 
>>286317
Срать нужно в сортир. Понято?
Аноним 22/04/16 Птн 11:50:05 #471 №286323 
>>286318
>>286319
Это не вы в /d предлагали огурцов изгнать, борцы за расовую чистоту?
Аноним 22/04/16 Птн 11:51:45 #472 №286324 
>>286323
Себя изгони, быдло.
Аноним 22/04/16 Птн 13:16:19 #473 №286348 
>>286311
>Стоит вообще создать отдельный коспирологического бугурта тред.
Не стоИт. Есть раздел для конспирологов, сиди, бугурти там, это говно сюда носить не надо.
Аноним 22/04/16 Птн 13:28:01 #474 №286354 
>>286305
какие вы нежные
просто так работают человеческие мозги, и ты проявляешь туже реакцию что и они. Считай что они играют в подводников в толчке, как маленький мальчик.
Какая разница во что они играют. Если ты думаешь что это заразит других темных - то это не так, у других другие игрушки. Вероятность что они чтото решают или чтото решат в этой области равна нулю.

В общем и целом, для человечества это хорошо - это дивергенция в подходах, которая помогает нам решать проблему. Правил нет, отбор все еще продолжается.

То что мы не можем договориться, это тоже проявление этого момента, иногда это хорошо, иногда не очень, но поскольку делают те кто хочет, а получается у тех кто прав - общий баланс положительный.
Аноним 22/04/16 Птн 15:17:50 #475 №286398 
>>285894
>Скорость света больше которой ничего не может быть.
Это условность, которую, безграмотные, со временем превратили в догму
Аноним 22/04/16 Птн 15:22:25 #476 №286401 
>>286398
Окей, мальчик, что тогда быстрее фотона?
Аноним 22/04/16 Птн 15:54:22 #477 №286414 
>>286401
Пожар в штанах Ацюковского, например.
Аноним 22/04/16 Птн 17:39:42 #478 №286468 
14613359822580.jpg
>>286414
>Пожар в штанах Ацюковского, например.
Шуточки шутим, гражданин. Ну а если серьезно - сможешь ли ты построить некое непротиворечивое описание мира без введения некой предельной константы-скорости (c)?
Аноним 22/04/16 Птн 17:42:29 #479 №286472 
>>286468
Делать мне нехуй что ли? Тут тред тупых вопросов, а не кружок теоретической физики.
Аноним 22/04/16 Птн 17:55:22 #480 №286483 
14613369228430.jpg
Что будет, если эту няшу закинуть в космос?
Аноним 22/04/16 Птн 18:02:16 #481 №286491 
14613373364810.jpg
Есть ли где-то я знаю, что есть, но где? расчёты по затратам условного топлива на вывод условного объекта на орбиту?

Уточню, что имею ввиду, сформулировав вопрос.

Сколько процентов топлива тратится на преодоление сопротивления атмосферы Земли? Положим, что у Земли бы не было атмосферы, то сколько бы процентов сэкономили при выводе объекта на орбиту?

Получив на ответ на этот вопрос, я смогу в моменты скуки фантазировать о старте ракеты с платформы на воздушных шариках. Ведь представь, на высоте 8 км плотность атмосферы в 2 раза меньше плотности атмосферы у поверхности, а это значит, что, подняв ракету на воздушных шариках до высоты Эвереста, мы получим следующие: 0,5 А + П = затраты на вывод, где А - топливо потраченное из-за сопротивления атмосферы, а П - топливо, потраченное из-за силы притяжения Земли при обычном космодромном старте. Допустим, П соотносится с А, как 1 к 1, тогда, мы имеем экономию в 25%!!!
Аноним 22/04/16 Птн 18:18:48 #482 №286509 
14613383283210.jpg
>>286483
сливкошоу?
Аноним 22/04/16 Птн 18:43:22 #483 №286543 
>>286509
Ага, делал по их мануалу.
Аноним 22/04/16 Птн 19:04:14 #484 №286552 
>>286483
Помрет от перегрева, или замерзает нахуй, или убьёт нахуй радиацией.
Аноним 22/04/16 Птн 19:04:52 #485 №286553 
>>286491
Обычно при выводе на низкую орбиту Земли тратится в районе 9.5 км/с dV, при этом орбитальная скорость примерно 8 км/с. Разница 1.5 км/с, то есть порядка 15% dV.
Аноним 22/04/16 Птн 20:52:27 #486 №286594 
>>286491
> потраченного топлива
Кто потраченный, ты потраченный, ёпт. Не я потраченный, не он, а ты потраченный.
Забудь всю эту хуергу что ты написал. При описании РН или КА как цельной системы никто не измеряет топливо в литрах, килограммах или процентах. Иди изучай уравнение Циолковского для многоступенчатой ракеты, и что такое характеристическая скорость, она же Δv, она же просто дельта. Это универсальная метрика энергетических возможностей аппарата независимо от его размеров и массы. В случае атмосферного полёта применимость дельты ограничена, но для РН почти похуй.

> Положим, что у Земли бы не было атмосферы, то сколько бы процентов сэкономили при выводе объекта на орбиту?
Мало, т.к. ракета и так быстро проходит атмосферу, основной разгон вбок происходит практически в вакууме. Атмосферные потери для большинства РН малозначительны, и чем больше ракета - тем больше на них похуй.

Для примера у Сатурна 5 атмосферные потери составляли всего лишь 40м/с, у малых-средних ракет типичная цифра - 100-150м/с. Ощутимо, но по сравнению с гравитационными потерями - хуйня. Даже потери на не-экваториальность космодрома при запуске на экваториальную орбиту и то больше.

Атмосферные потери имеют какое-то значение лишь в миллипиздрических ракетках, выводящих меньше 100кг на орбиту.
Аноним 22/04/16 Птн 21:15:33 #487 №286605 
>>286594
>Атмосферные потери имеют какое-то значение лишь в миллипиздрических ракетках, выводящих меньше 100кг на орбиту.

Но ведь еще и потеря импульса движков от внешнего давления.

мимодругойанон
Аноним 22/04/16 Птн 21:42:51 #488 №286615 
>>286605
Так и гравитационные потери тоже нихуевые, а масса быстро растёт, согласно дедушке Ц. Но даже если облегчить ракету раза в 2, смысла аэростатному пуску это не прибавит.
Аноним 22/04/16 Птн 22:16:56 #489 №286622 
Есть видос как космонавты надевают свои скафандры? Как Армстронг в свой влезал?
Только не говорите что их в скафандры еще на заводе впихивают и они все время в них ходят от старта до приземления
Аноним 22/04/16 Птн 22:35:49 #490 №286626 
>>286622
Скафандры разные бывают и надевают их по-разному. Например Сокол надевают как комбинезон, американский EMU состоит из двух половинок, верхней и нижний. A7L, который использовали для миссий Аполлонов тоже заебно одевается. Проще всего напялить Орлан, там тупо дверь в спине.
Аноним 22/04/16 Птн 22:37:35 #491 №286627 
>>286622
ссылку забыл
https://www.youtube.com/results?search_query=how+to+put+on+a+space+suit

Аноним 23/04/16 Суб 10:49:35 #492 №286753 
Правда, что капсулы союзов при возвращении на землю заметно фонят?
Неужели это от солнечной радиации?
Аноним 23/04/16 Суб 10:52:16 #493 №286754 
>>286753
Те, кто мерял не скажут нихуя, пиздёж это всё.
Аноним 23/04/16 Суб 10:58:44 #494 №286756 
>>286753
На дне союза есть датчик, который действительно излучает небольшие дозы радиации. Но он фонит направленно, а не во все стороны, так что к нему просто не подходят с той стороны.
Аноним 23/04/16 Суб 14:28:32 #495 №286837 
>>286753
Гамма-лучевой высотомер там используется (Кактус), для точного определения когда запускать реактивный тормоз. Он содержит в себе немного изотопов и чуть-чуть фонит, ничего страшного даже если под луч попадешь (а он выключается после посадки к тому же). Не сильнее чем рентген-фотка получится, ну и если Союз разъебется, то источник из ошметков лучше руками не хватать, а сначала посмотреть дозиметром. Но там СТРАШНЫЙ И УЖАСНЫЙ ЗНАК РАДИАЦИОННОЙ ОПАСНОСТИ ЧЕРНОБЫЛЬ СЛОНОВЬЯ НОГА ДВУХМЕТРОВЫЕ КРЫСОМУТАНТЫ ГРОБ ПИДОР, поэтому ходит такая байка.
Аноним 23/04/16 Суб 16:01:03 #496 №286877 
>>286837
ИДИ СВОЕЙ ДОРОГОЙ СТАЛКЕР
Аноним 23/04/16 Суб 17:29:33 #497 №286921 
>>286837
>он выключается после посадки
>немного изотопов и чуть-чуть фонит
может тогда его и при взлете отключать и под командиром изоляция не нужна - оптимизация хуле.
Аноним 23/04/16 Суб 17:41:59 #498 №286924 
14614225196880.jpg
>>286921
Так несильно фонит из-за того, что там изоляция есть. Если оболочку убрать, у экипажа будут трехголовые дети, понятное дело, тут как и с ритэгами. Тащемта шторка, перекрывающая поток, там предусмотрена в любом случае, насколько я понимаю.
Аноним 23/04/16 Суб 18:44:25 #499 №286947 
>>286924
меня выключать смутило, но конечно пластина свинца тоже выключатель, просто подумал что кто то потек.
Аноним 23/04/16 Суб 20:07:16 #500 №286978 
>>286270
бамп вопросу
Аноним 23/04/16 Суб 21:16:55 #501 №287005 
>>286924
Зачем был нужен именно гамма-лучевой высотомер? Почему радио не подошло?
или щуп десятиметровый
Аноним 23/04/16 Суб 21:48:27 #502 №287020 
>>287005
Хуй знает. Вероятно максимум точность и неприхотливость к погоде/поверхности внизу. Они же на петардах садятся.
Аноним 23/04/16 Суб 21:51:25 #503 №287021 
>>287020
В комментариях к видеороликам с посадками американцы траллируют. Говорят, русские ебанулись с разбегу садиться - аж пыль столбом. Как они языки себе не откусывают? Вот у нас были шаттлы - они себе такого не позволяли. Ну, то ж русские...
Аноним 23/04/16 Суб 21:54:29 #504 №287026 
>>286978
>>286994
Аноним 23/04/16 Суб 23:13:16 #505 №287083 
14614423961930.jpg
>>287021
>русские ебанулись с разбегу садиться - аж пыль столбом.
Аноним 24/04/16 Вск 02:05:56 #506 №287150 
Какие основные сложности прямо сейчас запустить ракету и прибуксировать астероид на орбиту? Дорого ли?
Дирижабль, ага. Аноним 24/04/16 Вск 03:04:38 #507 №287160 
Я так понял аэростаты поднимаются на 50 км.
Если нам надо вывести на НОО то это уже четверть пути.
Разве эти 50 км мало?
Аноним 24/04/16 Вск 10:53:44 #508 №287182 
>>287160
Это дороже и технически сложнее. Размер аэростата посчитай. Плюс нетривиальность задачи по удержанию всей системы в нужном положении.
Аноним 24/04/16 Вск 11:18:08 #509 №287184 
>>287160
Мало. Потому что надо будет набрать ещё 7.2 м/с. Даже ссто на фоне этого омска выглядит образцом практичности
Аноним 24/04/16 Вск 12:14:54 #510 №287207 
14614892940320.jpg
Почему галактики должны разрушиться?
Аноним 24/04/16 Вск 12:29:52 #511 №287213 
>>287207
Уноси это говно.
Аноним 24/04/16 Вск 13:09:08 #512 №287228 
>>287207
Кому должны? Большой разрыв - одна из неподтвержденных космологических теорий о далеком будущем вселенной. Собственно в одноименной статье на википедии ты найдешь все, что я мог бы тебе сейчас сказать.
>>287213
Че несешь? Тебе картинка не понравилась? Кефирщик штоле?
Аноним 24/04/16 Вск 14:29:09 #513 №287246 
>>287228
Не кому, а почему.
Аноним 24/04/16 Вск 14:30:36 #514 №287247 
>>286626
>Проще всего напялить Орлан, там тупо дверь в спине.
Заходишь, а там тебе говорят: "Я не Олег". Или: "Вы там охуели штоле!". Вот смеху-то!
Аноним 24/04/16 Вск 14:37:18 #515 №287248 
>>286754
Этот прав:
>Те, кто мерял не скажут нихуя, пиздёж это всё.
В нашей космической отрасли - секретность дохуя, никто ничего не скажет. Они как военные: даже женам - ни-ни.
Аноним 24/04/16 Вск 14:58:15 #516 №287253 
14614990956690.jpg
>>287247
Расхохотался. Вот как выглядело при этом мое лицо.
Аноним 24/04/16 Вск 15:57:59 #517 №287267 
>>287228
> Че несешь?
Там таймлайн проёбан, сама сама шкала ебанутая, да и манятеории это всё. Алсо галактики не разрушаются, а падает звездообразование и, как следствие - светимость.
Аноним 24/04/16 Вск 17:09:37 #518 №287296 
14615069774740.gif
Допустим по Луне скрытно едет колонна танков Т-72 с герметичными люками. Их будет слышно через сейсмограф с достаточной точностью, чтобы определить направление и расстояние?
Аноним 24/04/16 Вск 17:11:32 #519 №287297 
В космосе можно буксировать тросом?
Аноним 24/04/16 Вск 17:11:48 #520 №287299 
Чому на Юпитер, Сатурн летают кучи зондов а Уран и Нептун никто кроме Вояджера-2 не посетил?
И даже не планируют посещать...
Даже на ебаный планетоид-плутон слетали, а на газовые гиганты не могут.
А там много интересного. Спутники всякие, много еще чего.

Почему даже проектов нет по полетам к Нептуну например?
Аноним 24/04/16 Вск 17:14:38 #521 №287301 
>>287267
>Там таймлайн проёбан, сама сама шкала ебанутая
Схемка для тех, кто попроще, очевидно.
>манятеории
Что не манятеории? Биг рип вполне себе принимается как один из возможных вариантов. Развитие технологий или методологии пока недостаточное, чтобы однозначно опровергнуть или подтвердить.
>Алсо галактики не разрушаются
Ээм, ты вообще с теорией большого разрыва знаком?
Аноним 24/04/16 Вск 17:17:06 #522 №287302 
>>287299
Юпитер - самая большая доступная планетная йоба в системе, у Сатурна такие-то кольца. Проще изучать их, а на тех недомерков экстраполировать данные. Уран в общем-то тоже тот еще фрукт, лежит криво и кольца у него вертикальные, но просто до него и Нептуна дальше и сложнее летать, вот и весь секрет.
Аноним 24/04/16 Вск 17:18:59 #523 №287305 
14615075397650.jpg
допустим, построил я на орбите космическую станцию. и решил назвать например орионом. для большей символичности, решил поставить на орбите в таком положении, чтобы орбитальная станция орион всегда находилась в том же месте, что созвездие орион на относительно земли.
как такого добиться на низкой орбите? на высокой орбите? какова должна быть высота орбиты для осуществления задумки?
Аноним 24/04/16 Вск 17:20:40 #524 №287307 
>>287302
Ну они рили другие. И атмосфера, и состав. И холоднее они. Уран вообще странно холодный.
И спутников там просто дохуя - и у того, и у того. А возможно что езе больше только не открыты они.
У миранды сильная геологическая активность например.

ИМХО знаний от исследования системы Урана или Нептуна будет больше чем полет в сторону Плутона и дальнейший пересчет камушков за границами планетарной системы.
Аноним 24/04/16 Вск 17:23:24 #525 №287314 
14615078040910.jpg
Мы в бамплимите, сейчас запилю тред и перенесете некоторые свои вопросы туда.
Аноним 24/04/16 Вск 17:30:34 #526 №287319 
Вики пишет что у нептуна окна для запуска с интервалом в 12 лет идут.
В 2019 закроется и хуй соси до 30-х.
Реально за 4 года запилить проект, зонд и ракету для полета к Нептуну?
Аноним 24/04/16 Вск 17:40:13 #527 №287321 
>>287253
Олег штоле?
Аноним 24/04/16 Вск 17:47:21 #528 №287331 
Перекат

https://2ch.pm/spc/res/287328.html

https://2ch.pm/spc/res/287328.html

https://2ch.pm/spc/res/287328.html

https://2ch.pm/spc/res/287328.html
Аноним 24/04/16 Вск 18:30:34 #529 №287353 
>>287305
Если я правильно тебя понял, то на орбите, гравитационносвязанной с Землей, такого положения добиться нельзя.
Аноним 24/04/16 Вск 22:45:19 #530 №287522 
>>287305
Че за матан на пике? Меня глючит или это прикол?
Аноним 24/04/16 Вск 22:46:57 #531 №287524 
>>287522
http://ryb.ru/2016/04/20/287077
Аноним 24/04/16 Вск 22:51:10 #532 №287533 
>>287524
Я прсто в ахуе. Даже нечего сказать.
Аноним 30/04/16 Суб 17:24:25 #533 №290900 
>>281033
Медленно замерзать будет, быстрее разлетится в мелкодисперсную пыль и замёрзнет в такой форме.
Аноним 11/05/16 Срд 21:58:39 #534 №293872 
>>287319
Да. Взять например проект новых горизонтов, там даже перепиливать много не надо. Но зачем?
comments powered by Disqus

Отзывы и предложения